Cardio Dalisay

Lakukan tugas rumah & ujian kamu dengan baik sekarang menggunakan Quizwiz!

The treatment for cardiac failure is directed at: a. decreasing oxygen needs of the heart. b. increasing the cardiac output by strengthening muscle contraction or decreasing peripheral resistance. c. reducing the amount of circulating blood volume. d. all of the above.

. d. all of the above.

Hypoxic spells in the infant with CHD can cause which of the following? Select all that apply. 1. Polycythemia. 2. Blood clots. 3. CVA. 4. Developmental delays. 5. Viral pericarditis. 6. Brain damage. 7. Alkalosis.

1. Polycythemia. 2. Blood clots. 3. CVA. 4. Developmental delays. 6. Brain damage.

A 3-month-old has been diagnosed with a VSD. The flow of blood through the heart with this type of defect is: 1. Right to left. 2. Equal between the two chambers. 3. Left to right. 4. Bypassing the defect.

1. Right to left.

. A client is admitted with a diagnosis of acute infective endocarditis (IE). Which findings during a nursing assessment support this diagnosis? SELECT ALL THAT APPLY. 1. Skin petechiae 2. Crackles in lung bases 3. Peripheral edema 4. Murmur 5. Arthralgia 6. Decreased erythrocyte sedimentation rate (ESR)

1. Skin petechiae 2. Crackles in lung bases 3. Peripheral edema 4. Murmur 5. Arthralgia

BP screenings to detect end-organ damage should be done routinely beginning at what age? 1. Birth. 2. 3 years. 3. 8 years. 4. 13 years.

2. 3 years.

. A nurse increases activity for a client with an admit ting diagnosis of acute coronary syndrome. Which symptoms experienced by the client best support a nursing diagnosis of activity intolerance? 1. Pulse rate increased by 15 beats per minute during activity 2. Blood pressure (BP) 130/86 mm Hg before activity; BP 108/66 mm Hg during activity 3. Increased dyspnea and diaphoresis relieved when sitting in a chair 4. A mean arterial pressure (MAP) of 80 following Activity

2. Blood pressure (BP) 130/86 mm Hg before activity; BP 108/66 mm Hg during activity

Family discharge teaching has been effective when the parents of a female toddler diagnosed with KD states: 1. "The arthritis in her knees is permanent. She will need knee replacements." 2. "I will give her a pain reliever for her peeling palms and soles of her feet." 3. "I know she will be irritable for 2 months after her symptoms started." 4. "I will continue with high doses of Tylenol for her inflammation."

3. "I know she will be irritable for 2 months after her symptoms started."

The nurse in the pediatric telemetry unit has been reviewing heart rhythms in children. The most common dysrhythmia in pediatrics is: 1. Ventricular tachycardia. 2. Sinus bradycardia. 3. Supraventricular tachycardia. 4. First-degree heart block.

3. Supraventricular tachycardia.

A client is hospitalized for heart failure secondary to alcohol-induced cardiomyopathy. The client is started on milrinone (Primacor®) and placed on a transplant waiting list. The client has been curt and verbally aggressive in expressing dissatisfaction with the med ication orders, overall care, and the need for energy conservation. A nurse should interpret that the client's behavior is likely related to the client's: 1. denial of the illness. 2. reaction to milrinone (Primacor®). 3. fear of the diagnosis. 4. response to cerebral anoxia.

3. fear of the diagnosis.

A nurse reviews a clients laboratory results. Which findings should alert the nurse to the possibility of atherosclerosis? (Select all that apply.) a. Total cholesterol: 280 mg/dL b. High-density lipoprotein cholesterol: 50 mg/dL c. Triglycerides: 200 mg/dL d. Serum albumin: 4 g/dL e. Low-density lipoprotein cholesterol: 160 mg/dL

ACE a. Total cholesterol: 280 mg/dL c. Triglycerides: 200 mg/dL e. Low-density lipoprotein cholesterol: 160 mg/dL

A nurse cares for a client who is recovering from a right-sided heart catheterization. For which complications of this procedure should the nurse assess? (Select all that apply.) a. Thrombophlebitis b. Stroke c. Pulmonary embolism d. Myocardial infarction e. Cardiac tamponade

ACE. a. Thrombophlebitis c. Pulmonary embolism e. Cardiac tamponade

A nurse receives a serum laboratory report for six different clients with admitting diagnoses of chest pain. After reviewing all of the lab reports, in which order should the nurse address each lab value? Priori tize the order in which the nurse should address each of the clients' results. ______ Troponin T 42 ng/mL (0.0-0.4 ng/mL) ______ WBC 11,000 K/µL ______ Hgb 7.2 g/dL ______ SCr 2.2 mg/dL ______ K 2.2 mEq/L ______ Total cholesterol 430 mg/dL

Answer: 1, 6, 3, 4, 2, 5

An emergency room nurse assesses a female client. Which assessment findings should alert the nurse to request a prescription for an electrocardiogram? (Select all that apply.) a. Hypertension b. Fatigue despite adequate rest c. Indigestion d. Abdominal pain e. Shortness of breath

BCE. b. Fatigue despite adequate rest c. Indigestion e. Shortness of breath

A nurse assesses a client who is recovering after a coronary catheterization. Which assessment findings in the first few hours after the procedure require immediate action by the nurse? (Select all that apply.) a. Blood pressure of 140/88 mm Hg b. Serum potassium of 2.9 mEq/L c. Warmth and redness at the site d. Expanding groin hematoma e. Rhythm changes on the cardiac monitor

BDE b. Serum potassium of 2.9 mEq/L d. Expanding groin hematoma e. Rhythm changes on the cardiac monitor

A nurse prepares a client for a pharmacologic stress echocardiogram. Which actions should the nurse take when preparing this client for the procedure? (Select all that apply.) a. Assist the provider to place a central venous access device. b. Prepare for continuous blood pressure and pulse monitoring. c. Administer the clients prescribed beta blocker. d. Give the client nothing by mouth 3 to 6 hours before the procedure. e. Explain to the client that dobutamine will simulate exercise for this examination.

BDE. b. Prepare for continuous blood pressure and pulse monitoring. d. Give the client nothing by mouth 3 to 6 hours before the procedure. e. Explain to the client that dobutamine will simulate exercise for this examination.

The nurse is caring for a 9-month-old who was born with a CHD. Assessment reveals an HR of 160, capillary refill of 4 seconds, bilateral crackles, and sweat on the scalp. These are signs of ___________________.

CHF

TEST BANK BRUNNER !!!

DRINK YOUR WATER BITCH <333

The ______________ serves as the septal opening between the atria of the fetal heart.

FORAMEN OVALE

IGNATIVICIUS !!!

HEHE

TEST BANK DAVIS !!!

HINGA MUNA <3333

The nurse is caring for a 3-year-old boy whose caregiver noticed that his eyes are reddened with no discharge, and his palms and soles of the feet are red, swollen, and peeling. Upon examination, the nurse's assessment includes dry, cracked lips and a "strawberry tongue." The nurse most likely suspects ______________________________.

KD

. Indomethacin may be given to close what CHD in newborns?

PDA

A newborn is diagnosed with a CHD. The test results reveal that the lumen of the duct between the aorta and pulmonary artery remains open. This defect is known as __________

PDA.

For the child with hypoplastic left heart syndrome, what drug may be given to allow the PDA to remain open until surgery?

PROSTAGLANDIN E

TEST BANK PEDIA SUCCESS !!!

SHINING BRIGHT LIKE A DIA MOND

The 6-month-old who has a "tet spell" could have the CHD defect of decreased pulmonary blood flow called ___________________________

TOF

A nurse is assessing an older adult admitted with a diagnosis of left-sided heart failure and mitral regur gitation. Identify the area with an X where the nurse should place the stethoscope to best auscultate the murmur associated with mitral regurgitation.

The mitral valve is best heard with the bell of the stethoscope at the fifth intercostal space, left midclavicular line. The bell is used to aus cultate low-pitched sounds. Abnormalities, such as S3 or S4, are best heard with the bell of the stethoscope.

. A 10-year-old child is recovering from a severe sore throat. The caregiver now states that the child complains of chest pain. The nurse observes that the child has swollen joints, nodules on the fingers, and a rash on the chest. The likely cause of this syndrome is _________

To make the diagnosis of RF, major and minor criteria are used. Major criteria include carditis, subcutaneous nodules, erythema marginatum, chorea, and arthritis. Minor criteria include fever and previous history of RF

An example of a potassium-sparing diuretic that might be prescribed for a person with congestive heart failure is: a. Aldactone. b. Mykrox. c. Zaroxolyn. d. Lasix.

a. Aldactone.

. A client had a femoropopliteal bypass graft with a synthetic graft. What action by the nurse is most important to prevent wound infection? a. Appropriate hand hygiene before giving care b. Assessing the clients temperature every 4 hours c. Clean technique when changing dressings d. Monitoring the clients daily white blood cell count

a. Appropriate hand hygiene before giving care

Probably the strongest risk factor for the development of atherosclerotic lesions is: a. cigarette smoking. b. lack of exercise. c. obesity. d. stress.

a. cigarette smoking.

With ventricular tachycardia: a. conduction originates in the ventricle. b. electrical defibrillation is used immediately. c. the P wave usually is normal. d. the ventricular rate is twice the normal atrial rate.

a. conduction originates in the ventricle.

Rheumatic endocarditis is an inflammatory reaction to: a. group A, beta-hemolytic streptococcus. b. Pseudomonas aeruginosa. c. Serratia marcescens. d. Staphylococcus aureus.

a. group A, beta-hemolytic streptococcus.

Identify three drug classifications that are commonly prescribed for management or treatment: _______________, _________________, and ______________________.

analgesics, antibiotics, and corticosteroids

A nursing student learns about modifiable risk factors for coronary artery disease. Which factors does this include? (Select all that apply.) a. Age b. Hypertension c. Obesity d. Smoking e. Stress

b. Hypertension c. Obesity d. Smoking e. Stress

A nurse is caring for a client with a deep vein thrombosis (DVT). What nursing assessment indicates a priority outcome has been met? a. Ambulates with assistance b. Oxygen saturation of 98% c. Pain of 2/10 after medication d. Verbalizing risk factors

b. Oxygen saturation of 98%

Backward flow of blood from the left ventricle to the left atrium is through the: a. aortic valve. b. mitral valve. c. pulmonic valve. d. tricuspid valve.

b. mitral valve.

Xenographs, used for valve replacement, have a viability of about: a. 2 years. b. 4 years. c. 8 years. d. 12 years.

c. 8 years.

Ventricular rate and rhythm can be determined by examining what interval on an ECG strip? a. PP interval b. QT interval c. RR interval d. TP interval

c. RR interval

Atrioventricular (AV) nodal reentry tachycardia is characterized by an atrial rate: a. of 100 bpm. b. between 100 and 150 bpm. c. between 150 and 250 bpm. d. more than 250 bpm.

c. between 150 and 250 bpm.

The nurse knows that a patient who is to receive a xenograft from a pig or cow will be receiving a (an): a. allograft. b. autograft. c. heterograft. d. homograft.

c. heterograft.

On assessment, the nurse knows that the presence of pitting edema indicates fluid retention of at least: a. 4 lb. b. 6 lb. c. 8 lb. d. 10 lb.

d. 10 lb.

Ventricular fibrillation is associated with an absence of: a. heartbeat. b. palpable pulse. c. respirations. d. all of the above.

d. all of the above.

For discharge planning, Mr. Lillis is advised to: a. avoid large meals. b. exercise daily. c. restrict caffeine-containing beverages. d. do all of the above.

d. do all of the above.

On the basis of assessment data, the physician diagnoses an acute myocardial infarction. List the drug classification that the nurse knows should be given within 3 to 6 hours of diagnosis: ______________________. List two common examples: ________________ and _____________________.

thrombolytics; alteplase, and reteplase

The primary objective of medical management is: _______________________________________________. .

total eradication of the invading organism

A 16-year-old male is diagnosed with hypertension. His laboratory values are hemoglobin B 16 g/dL, hematocrit level 43%, sodium 139 mEq/L, potassium 4.4 mEq/L, and total cholesterol of 220 mg/dL. Which of the following drugs would increase his total cholesterol? 1. Beta blockers. 2. Calcium channel blockers. 3. ACE inhibitors. 4. Diuretics.

1. Beta blockers.

The nurse is caring for a preschool female diagnosed with CHF. She is receiving maintenance doses of digoxin and furosemide. She is rubbing her eyes when she is looking at the lights in the room, and her HR is 70 beats per minute. The nurse suspects which laboratory finding? 1. Hypokalemia. 2. Hypomagnesemia . 3. Hypocalcemia. 4. Hypophosphatemia.

1. Hypokalemia.

The nurse is caring for a school-aged boy with KD. A student nurse who is on the unit asks if there are medications to treat this disease. The nurse's response to the student nurse is: 1. Immunoglobulin G and aspirin. 2. Immunoglobulin G and ACE inhibitors. 3. Immunoglobulin E and heparin. 4. Immunoglobulin E and ibuprofen

1. Immunoglobulin G and aspirin.

The following are examples of acquired heart disease. Select all that apply. 1. Infective endocarditis 2. Hypoplastic left heart syndrome. 3. RF. 4. Cardiomyopathy. 5. KD. 6. Transposition of the great vessels.

1. Infective endocarditis 3. RF. 4. Cardiomyopathy. 5. KD.

A child with a history of cardiac surgery requires an annual electrocardiogram. What can the electrocardiogram detect? Select all that apply. 1. Ischemia. 2. Injury. 3. CO. 4. Dysrhythmias 5. SVR. 6. Occlusion pressure. 7. Conduction delay

1. Ischemia. 2. Injury. 4. Dysrhythmias 7. Conduction delay

The parent of an infant newly diagnosed with TOF is asking the nurse which defects are involved. Select all that apply. 1. VSD. 2. Right ventricular hypertrophy. 3. Left ventricular hypertrophy. 4. PS. 5. Pulmonic atresia. 6. Overriding aorta. 7. PDA.

1. VSD. 2. Right ventricular hypertrophy. 4. PS. 6. Overriding aorta.

A nurse is to administer 40 mg of furosemide (Lasix®) to a client in heart failure. The prefilled syringe reads 100 mg/mL. In order to give the correct dose, the nurse should administer ____mL to the client.

100 mg: 1 mL :: 40 mg : X mL 100 X = 40 X = 0.4 0.4 mL should be administered to the client.

The nurse is caring for a 4-year-old female with a Glasgow Coma Scale of 3, HR of 88 beats per minute and regular, respiratory rate of 22, BP of 78/52, and blood sugar of 35 mg/dL. The nurse asks the caregiver about accidental ingestion of what drug? 1. Calcium channel blocker. 2. Beta blocker. 3. ACE inhibiter. 4. ARB.

2. Beta blocker.

The school nurse has been following an 8-year-old female who comes to the office frequently. She has come mainly for vague complaints of dizziness and headache. Today, she is brought after fainting in the cafeteria following a nosebleed. Her BP is 122/85, and her radial pulses are bounding. Calling for the ambulance, the nurse suspect she has: 1. Transposition of the great vessels. 2. COA. 3. AS. 4. PS.

2. COA.

Which of the following assessments indicate that the parents of a 7-year-old are following the prescribed treatment for CHF? 1. HR of 56 beats per minute. 2. Elevated red blood cell count. 3. 50th percentile height and weight for age. 4. Urine output of 0.5 cc/kg/hr

3. 50th percentile height and weight for age.

A 15-year-old female who is sexually active is diagnosed with secondary hypertension. She admits to intermittent use of birth control. Which of the following drugs should not be used to control her BP? 1. Beta blockers. 2. Calcium channel blockers. 3. ACE inhibitors. 4. Diuretics.

3. ACE inhibitors.

Which medication should the nurse give to a patient who is diagnosed with transposition of the great vessels? 1. Ibuprofen. 2. Betamethasone. 3. Prostaglandin E. 4. Indocin.

3. Prostaglandin E.

After an inferior-septal wall myocardial infarction, which complication should a nurse suspect when noting jugular venous distention (JVD) and ascites? 1. Left-sided heart failure 2. Pulmonic valve malfunction 3. Right-sided heart failure 4. Ruptured septum

3. Right-sided heart failure

Which statement by the mother of an infant boy with CHF who is being sent home on digoxin indicates she needs further education on the care of her child? 1. "I will give him the medication at regular 12-hour intervals." 2. "If he vomits, I will not give him a make-up dose." 3. "If I miss a dose, I will not give an extra dose, but keep him on his same schedule." 4. "I will mix the digoxin in some of his formula to make it taste better for him."

4. "I will mix the digoxin in some of his formula to make it taste better for him."

An infant has been diagnosed with an ASD, or AVC defect. The flow of blood through the heart with this type of defect is: 1. Right to left. 2. Equal between the two chambers. 3. Bypassing the defect. 4. In either direction.

4. In either direction.

A nurse prepares a client with acute renal insufficiency for a cardiac catheterization. The provider prescribes 0.9% normal saline to infuse at 125 mL/hr for renal protection. The nurse obtains gravity tubing with a drip rate of 15 drops/mL. At what rate (drops/min) should the nurse infuse the fluids? (Record your answer using a whole number, and rounding to the nearest drop.) _____ drops/min

ANS: 31 drops/min

A healthy level of serum cholesterol would be a reading of: a. 160 to 190 mg/dL. b. 210 to 240 mg/dL. c. 250 to 275 mg/dL. d. 280 to 300 mg/dL.

a. 160 to 190 mg/dL.

. Mr. Lillis needs to be advised that myocardial healing will not be complete for about: a. 2 months. b. 4 months. c. 6 months. d. 8 months.

a. 2 months.

A goal of dilation in PTCA is to increase blood flow through the artery's lumen and achieve a residual stenosis of less than: a. 20%. b. 35%. c. 60%. d. 80%

a. 20%.

The nurse advises a patient that sublingual nitroglycerin should alleviate angina pain within: a. 3 to 4 minutes. b. 10 to 15 minutes. c. 30 minutes. d. 60 minutes.

a. 3 to 4 minutes.

The characteristic sign of pericarditis is: a. a friction rub. b. dyspnea. c. fever. d. hypoxia.

a. a friction rub.

The nurse knows that the hallmark symptom of peripheral arterial occlusion disease is: a. intermittent claudication. b. phlebothrombosis. c. postphlebitis syndrome. d. thrombophlebitis

a. intermittent claudication.

When administering heparin anticoagulant therapy, the nurse needs to monitor the clotting time to make certain that it is within the therapeutic range of: a. one to two times the normal control. b. two to three times the normal control. c. 3.5 times the normal control. d. 4.5 times the normal control

a. one to two times the normal control.

Hemodynamic monitoring by means of a multilumen pulmonary artery catheter can provide detailed information about: a. preload. b. afterload. c. cardiac output. d. all of the above.

a. preload.

A dysrhythmia common in normal hearts and described by patients as "my heart skipped a beat" is: a. premature atrial complex. b. atrial flutter. c. sinus tachycardia. d. ventricular fibrillation.

a. premature atrial complex.

The difference between the systolic and the diastolic pressure is called the: a. pulse pressure. b. auscultatory gap. c. pulse deficit. d. Korotkoff sound.

a. pulse pressure.

The PR interval on an ECG strip that reflects normal sinus rhythm would be between: a. 0.05 and 0.10 seconds. b. 0.12 and 0.20 seconds. c. 0.15 and 0.30 seconds. d. 0.25 and 0.40 seconds

b. 0.12 and 0.20 seconds.

A commonly prescribed diuretic that is given intravenously to produce a rapid diuretic effect is: a. Bumex. b. Lasix. c. Mykrox. d. Zaroxolyn.

b. Lasix.

A nurse is assessing an obese client in the clinic for follow-up after an episode of deep vein thrombosis. The client has lost 20 pounds since the last visit. What action by the nurse is best? a. Ask if the weight loss was intended. b. Encourage a high-protein, high-fiber diet. c. Measure for new compression stockings. d. Review a 3-day food recall diary.

c. Measure for new compression stockings.

A client has an intra-arterial blood pressure monitoring line. The nurse notes bright red blood on the clients sheets. What action should the nurse perform first? a. Assess the insertion site. b. Change the clients sheets. c. Put on a pair of gloves

c. Put on a pair of gloves

The dominant function in cardiac failure is: a. ascites. b. hepatomegaly. c. inadequate tissue perfusion. d. nocturia.

c. inadequate tissue perfusion.

Pain is experienced when the arterial lumen narrows to about: a. 15%. b. 25%. c. 35%. d. 50%.

d. 50%.

Additional symptoms to support the nurse's diagnosis include all of the following except: a. blanched skin appearance when the limb is dependent. b. diminished distal pulsations. c. reddish-blue discoloration of the limb when it is elevated. d. warm and rosy coloration of the extremity after exercise.

d. warm and rosy coloration of the extremity after exercise.

Name the anatomic landmarks used to auscultate for a pericardial friction rub: ____________________________________________

left sternal edge in the fourth intercostal space

The pain associated with this condition commonly occurs in muscle groups _________________________. .

one joint level below the stenosis or occlusion

Based on assessment data, the major nursing diagnosis is: _________________________________________.

pain related to inflammation of the pericardium

The nurse notices that several minutes after Jack's leg is dependent, the vessels remain dilated. This is evidenced by the coloring of the skin, which the nurse describes as: a. rosy. b. rubor. c. pallor. d. cyanotic.

rubor

Mr. Fontana, a 60-year-old executive, is admitted to the hospital with a diagnosis of infective endocarditis. Pertinent history includes a previous diagnosis of mitral valve prolapse. A physical examination at his physician's office before admission reveals complaints of anorexia, joint pain, intermittent fever, and a 10-lb weight loss in the past 2 months. 1. The nurse knows, prior to assessment, that Mr. Fontana's vague clinical symptoms are characteristic of an insidious disease onset that develops from one of three conditions: ______________________, ______________________, or ______________________.

toxicity of the infection, heart valve destruction, or embolization of fragments of vegetative growth on the heart

Which statement by the mother of a patient with RF shows she has an understanding of prevention in her other children? 1. "Whenever one of them gets a sore throat, I will give that child an antibiotic." 2. "There is no treatment. It must run its course." 3. "If their culture is positive for group A streptococcus, I will give them their antibiotic." 4. "If their culture is positive for staphylococcus A, I will give them their antibiotic."

1. "Whenever one of them gets a sore throat, I will give that child an antibiotic."

A client admitted with unstable angina is started on intravenous heparin and nitroglycerin. The client's chest pain resolves, and the client is weaned from the nitroglycerin. Noting that the client had a synthetic valve replacement for aortic stenosis 2 years ago, a physician writes an order to restart the oral warfarin (Coumadin®) 5 mg at 1900 hours. Which is the nurse's best action? 1. Administer the warfarin as prescribed. 2. Call the physician to question the warfarin order. 3. Discontinue the heparin drip and then administer the warfarin. 4. Hold the dose of warfarin until the heparin has been discontinued.

1. Administer the warfarin as prescribed.

. The nurse is caring for an infant with CHF. The following are interventions to decrease cardiac demands on the infant. Select all that apply. 1. Allow parents to hold and rock their child. 2. Feed only when the infant is crying. 3. Keep the child uncovered to promote low body temperature. 4. Make frequent position changes. 5. Feed the child when sucking the fists. 6. Change bed linens only when necessary. 7. Organize nursing activities.

1. Allow parents to hold and rock their child. 4. Make frequent position changes. 5. Feed the child when sucking the fists. 6. Change bed linens only when necessary. 7. Organize nursing activities.

Which nursing actions should a nurse plan when caring for a client experiencing dyspnea due to heart failure and chronic obstructive pulmonary disease (COPD)? SELECT ALL THAT APPLY. 1. Apply oxygen 6 liters per nasal cannula 2. Elevate the head of the bed 30 to 40 degrees 3. Weigh client daily in the morning 4. Teach client pursed-lip breathing techniques 5. Turn and reposition the client every 1 to 2 hours

1. Apply oxygen 6 liters per nasal cannula 2. Elevate the head of the bed 30 to 40 degrees 3. Weigh client daily in the morning 4. Teach client pursed-lip breathing techniques

A child born with Down syndrome should be evaluated for what associated cardiac manifestation? 1. CHD. 2. Systemic hypertension. 3. Hyperlipidemia. 4. Cardiomyopathy.

1. CHD

A 6-year-old is receiving aspirin therapy for KD. Exposure to what illnesses should be a cause to discontinue therapy and substitute dipyridamole (Persantine)? 1. Chickenpox or flu. 2. E. coli or staphylococcus. 3. Mumps or streptococcus A. 4. Streptococcus A or staphylococcus.

1. Chickenpox or flu.

A child has been diagnosed with KD. The parents are asking questions about the child's outcome. The nurse explains the most serious complications. Select all that apply. 1. Coronary thrombosis. 2. Coronary stenosis. 3. Coronary artery aneurysm. 4. Hypocoagulability. 5. Decreased sedimentation rate. 6. Hypoplastic left heart syndrome.

1. Coronary thrombosis. 2. Coronary stenosis. 3. Coronary artery aneurysm.

A nursing action that promotes ideal nutrition in an infant with CHF is: 1. Feeding formula that is supplemented with additional calories. 2. Allowing the infant to nurse at each breast for 20 minutes. 3. Providing large feedings every 5 hours. 4. Using firm nipples with small openings to slow feedings.

1. Feeding formula that is supplemented with additional calories.

At 0730 hours, a nurse receives a verbal order for a cardiac catheterization to be completed on a client at 1400 hours. Which action should the nurse initiate first? 1. Initiate NPO (nothing per mouth) status for the client. 2. Teach the client about the procedure. 3. Start an intravenous (IV) infusion of 0.9% NaCl. 4. Ask the client to sign a consent form.

1. Initiate NPO (nothing per mouth) status for the client.

A nurse is caring for a client following a coro nary artery bypass graft. Which assessment finding in the immediate postoperative period should be most concerning to the nurse? 1. No chest tube output for 1 hour when previously it was copious 2. Client temperature of 99.1°F (37.2°C) 3. Arterial blood gas (ABG) results show pH 7.32; Pco2 48; HCO3 28; Po2 80 4. Urine output of 160 mL in the last 4 hours

1. No chest tube output for 1 hour when previously it was copious

What two physiological changes occur as a result of hypoxemia in CHF? 1. Polycythemia and clubbing. 2. Anemia and barrel chest. 3. Increased white blood cells and low platelets. 4. Elevated erythrocyte sedimentation rate and peripheral edema

1. Polycythemia and clubbing.

A nurse should anticipate instructing a client scheduled for a coronary artery bypass graft to: SELECT ALL THAT APPLY. 1. discontinue taking aspirin prior to surgery. 2. perform postoperative cardiac rehabilitation exercises and stress management strategies. 3. wash with an antimicrobial soap the evening prior to surgery. 4. shave the chest and legs and then shower to remove the hair. 5. resume normal activities when discharged from the hospital. 6. expect close monitoring after surgery, several intravenous (IV) lines, a urinary catheter, endotracheal tube, and chest tubes.

1. discontinue taking aspirin prior to surgery. 2. perform postoperative cardiac rehabilitation exercises and stress management strategies. 3. wash with an antimicrobial soap the evening prior to surgery. 6. expect close monitoring after surgery, several intravenous (IV) lines, a urinary catheter, endotracheal tube, and chest tubes.

A nurse is planning care for a client admitted with a new diagnosis of persistent atrial fibrillation with rapid ventricular response. Although the client has had no previous cardiac problems, the client has been in atrial fibrillation for more than 2 days. The nurse should anticipate that the health-care provider is likely to initially order: SELECT ALL THAT APPLY. 1. oxygen. 2. immediate cardioversion. 3. administration of amiodarone (Cordarone®). 4. initiation of a IV heparin infusion. 5. immediate catheter-directed ablation of the AV node. 6. administration of a calcium channel antagonist such as diltiazem (Cardizem®).

1. oxygen. 3. administration of amiodarone (Cordarone®). 4. initiation of a IV heparin infusion. 6. administration of a calcium channel antagonist such as diltiazem (Cardizem®).

A nurse evaluates that a client understands dis charge teaching, following aortic valve replacement surgery with a synthetic valve, when the client states that he/she plans to: SELECT ALL THAT APPLY. 1. use a soft toothbrush for dental hygiene. 2. floss teeth daily to prevent plaque formation. 3. wear loose-fitting clothing to avoid friction on the sternal incision. 4. use an electric razor for shaving. 5. report black, tarry stools. 6. consume foods high in vitamin K, such as broccoli.

1. use a soft toothbrush for dental hygiene. 2. floss teeth daily to prevent plaque formation. 3. wear loose-fitting clothing to avoid friction on the sternal incision. 4. use an electric razor for shaving. 5. report black, tarry stools.

A client experiences cardiac arrest at home and is successfully resuscitated. Following placement of an implantable cardioverter-defibrillator (ICD), a nurse is evaluating the effectiveness of teaching for the client. Which statement, if made by the client, indicates that further teaching is needed? 1. "The ICD will monitor my heart activity and provide a shock to my heart if my heart goes into ventricular fibrillation again." 2. "When I feel the first shock I should tell my family to start cardiopulmonary resuscitation (CPR) and call 911." 3. "I am fearful of my first shock since my friend stated his shock felt like a blow to the chest." 4. "I will need to ask my physician when I can resume driving because some states disallow driving until there is a 6-month discharge-free period."

2. "When I feel the first shock I should tell my family to start cardiopulmonary resuscitation (CPR) and call 911."

The nurse is caring for a 3-month-old with a VSD. The physicians have decided not to repair it surgically. The parents express concern that this is not best for their child and ask why their daughter will not have an operation. The nurse's best response to the parents is: 1. "It is always helpful to get a second opinion about any serious condition like this." 2. "Your daughter's defect is small and will likely close on its own by the time she is 2 years old." 3. "It is common for the physicians to wait until an infant develops respiratory distress before they do the surgery because of the danger." 4. "With a small defect like this, we will wait until the child is 10 years old to do the surgery."

2. "Your daughter's defect is small and will likely close on its own by the time she is 2 years old."

The nurse is caring for a 1-year-old who has been diagnosed with CHF. Treatment began 3 days ago and has included digoxin and furosemide. The child no longer has retractions, lungs are clear and equal bilaterally, and HR is 96 beats per minute while the child sleeps. The nurse is confident that the child has diuresed successfully and has good renal perfusion when the nurse notes the child's urine output is: 1. 0.5 cc/kg/hr 2. 1 cc/kg/hr 3. 30 cc/hr 4. 1 oz/hr

2. 1 cc/kg/hr

A nurse is evaluating the blood pressure (BP) results for multiple clients with cardiac problems on a telemetry unit. Which BP reading suggests to the nurse that the client's mean arterial pressure (MAP) is abnormal and warrants notifying the physician? 1. 94/60 mm Hg 2. 98/36 mm Hg 3. 110/50 mm Hg 4. 140/78 mm Hg

2. 98/36 mm Hg

While assessing a newborn with respiratory distress, the nurse auscultates a machine-like heart murmur. Other findings are a wide pulse pressure, periods of apnea, increased PaCO2, and decreased PO2. The nurse suspects that the newborn has: 1. Pulmonary hypertension. 2. A PDA. 3. A VSD. 4. Bronchopulmonary dysplasia

2. A PDA.

Parents report that their 6-year-old has been seen by the school nurse for dizziness that occurred when standing in line for recess and homeroom since the start of the school term. The child now reports that she would rather sit and watch her friends play hopscotch because she cannot count out loud and jump at the same time. When the nurse asks the child if her chest ever hurts, she says yes. Based on this history, the nurse suspects that she has: 1. VSD. 2. AS. 3. Mitral valve prolapse. 4. Tricuspid atresia

2. AS.

Following a normal chest x-ray for a client who had cardiac surgery, a nurse receives an order to re move the chest tubes. Which intervention should the nurse plan to implement first? 1. Auscultate the client's lung sounds 2. Administer 4 mg morphine sulfate intravenously 3. Turn off the suction to the chest drainage system 4. Prepare the dressing supplies at the client's bedside

2. Administer 4 mg morphine sulfate intravenously

A client admitted with a diagnosis of acute coronary syndrome calls for a nurse after experiencing sharp chest pains that radiate to the left shoulder. The nurse notes, prior to entering the client's room, that the client's rhythm is sinus tachycardia with a 10-beat run of premature ventricular contractions (PVCs). Admitting orders included all of the following interventions for treating chest pain. Which should the nurse implement first? 1. Obtain a stat 12-lead electrocardiogram (ECG). 2. Administer oxygen by nasal cannula. 3. Administer sublingual nitroglycerin. 4. Administer morphine sulfate intravenously.

2. Administer oxygen by nasal cannula.

An infant with CHF is receiving digoxin to enhance myocardial function. What should the nurse assess prior to administering the medication? 1. Yellow sclera. 2. Apical pulse rate. 3. Cough. 4. Liver function test.

2. Apical pulse rate.

A 4-year-old is diagnosed with Wilm tumor. What associated manifestation does the nurse expect? 1. Atrial fibrillation. 2. Hypertension. 3. Endocarditis. 4. Hyperlipidemia.

2. Hypertension.

A child with a CHD undergoes the Norwood procedure. This procedure is used to correct: 1. Transposition of the great vessels. 2. Hypoplastic left heart syndrome. 3. TOF. 4. PDA.

2. Hypoplastic left heart syndrome.

. An 18-month-old with a myelomeningocele is going to undergo a cardiac catheterization. The mother expresses concern about the use of dye in the procedure. The child does not have any allergies listed on the medical record. In addition to the concern for an iodine allergy, what other allergy should the nurse bring to the attention of the catheterization staff? 1. Soy. 2. Latex. 3. Penicillin. 4. Dairy

2. Latex.

The mother of a toddler reports that the baby's father has just had a myocardial infarction. Because of this information, the nurse recommends the child have a(n): 1. Electrocardiogram. 2. Lipid profile. 3. Echocardiogram. 4. Cardiac catheterization.

2. Lipid profile.

While looking through the chart of an infant with a CHD of decreased pulmonary blood flow, the nurse would expect what laboratory finding? 1. Decreased platelet count. 2. Polycythemia. 3. Decreased ferritin level. 4. Shift to the left.

2. Polycythemia.

A 10-year-old has undergone a cardiac catheterization. At the end of the procedure, the nurse should first assess: 1. Pain. 2. Pulses. 3. Hemoglobin and hematocrit levels. 4. Catheterization report

2. Pulses.

The nurse is caring for an 8-year-old girl whose parents indicate she has developed spastic movements of her extremities and trunk, facial grimace, and speech disturbances. They state it seems worse when she is anxious and does not occur when she is sleeping. The nurse questions the parents about what recent illness? 1. KD. 2. RF. 3. Malignant hypertension. 4. Atrial fibrillation.

2. RF.

Aspirin has been ordered for the child with RF in order to: 1. Keep the PDA open. 2. Reduce joint inflammation. 3. Decrease swelling of strawberry tongue. 4. Treat ventricular hypertrophy of endocarditis.

2. Reduce joint inflammation.

A 1-year-old child is being prepared for a cardiac catheterization procedure. Which of the following findings about the child might delay the procedure? 1. 30th percentile for weight. 2. Severe diaper rash. 3. Allergy to soy. 4. Oxygen saturation of 91% on room air.

2. Severe diaper rash.

A nurse is teaching a client newly diagnosed with chronic stable angina. Which instructions should the nurse incorporate in the teaching session on measures to prevent future angina? SELECT ALL THAT APPLY. 1. Increase isometric arm exercises to build endurance. 2. Wear a face mask when outdoors in cold weather. 3. Take nitroglycerin before a stressful situation even though pain is not present. 4. Perform most exertional activities in the morning. 5. Avoid straining at stool. 6. Eliminate tobacco use.

2. Wear a face mask when outdoors in cold weather. 3. Take nitroglycerin before a stressful situation even though pain is not present. 5. Avoid straining at stool. 6. Eliminate tobacco use.

Because a step-down cardiac unit is unusually busy, a nurse fails to obtain vital signs at 0200 hours for a client 2 days postoperative for a mitral valve re placement. The client was stable when assessed at 0600 hours, so the nurse documents the electrocar diogram monitor's heart rate in the client's medical record for both the 0400 and 0600 vital signs. The charge nurse supervising the nurse determines that the nurse's behavior was: SELECT ALL THAT APPLY. 1. the correct action because neither complications nor harmful effects occurred. 2. a legal issue because the nurse has fraudulently falsified documentation. 3. demonstrating beneficence because the nurse decided what was best for the client. 4. an ethical issue of veracity because the nurse has been untruthful regarding the client's care. 5. an ethical legal issue of confidentiality because the nurse disclosed incorrect information. 6. demonstrating distributive justice because the nurse decided other clients' needs were priority

2. a legal issue because the nurse has fraudulently falsified documentation. 4. an ethical issue of veracity because the nurse has been untruthful regarding the client's care.

A nurse is caring for a client immediately following insertion of a permanent pacemaker via the right subclavian vein approach. The nurse best prevents pacemaker lead dislodgement by: 1. inspecting the incision site dressing for bleeding and the incision for approximation. 2. limiting the client's right arm activity and preventing the client reaching above shoulder level. 3. assisting the client with getting out of bed and ambulating with a walker. 4. ordering a stat chest x-ray following return from the implant procedure

2. limiting the client's right arm activity and preventing the client reaching above shoulder level.

. A nurse is working with a certified nursing assistant (CNA) providing care for four clients on a busy telemetry unit. All four clients are in need of immedi ate attention. The CNA is a senior nursing student who has been administering medications and per forming procedures during clinical experiences as a student nurse. The charge nurse supervising care on the telemetry unit determines that care is appropriate when the registered nurse (RN) working with the CNA delegates: SELECT ALL THAT APPLY. 1. administering acetaminophen (Tylenol®) to the client with an elevated temperature. 2. taking vital signs on the client newly admitted with a diagnosis of heart failure. 3. finishing the discharge instructions so the client with a new pacemaker implant can go home. 4. changing a client's chest tube dressing because it got wet when the water pitcher overturned. 5. providing a sponge bath for the client with the elevated temperature. 6. checking the lung sounds of the client whose chest tube drainage system was tipped over and then righted

2. taking vital signs on the client newly admitted with a diagnosis of heart failure. 5. providing a sponge bath for the client with the elevated temperature.

Which statement by the mother of a male toddler with RF shows she has good understanding of the care of her child? 1. "I will apply heat to his swollen joints to promote circulation." 2. "I will have him do gentle stretching exercises to prevent contractures." 3. "I will give him the aspirin that is ordered for pain and inflammation." 4. "I will apply cold packs to his swollen joints to reduce pain."

3. "I will give him the aspirin that is ordered for pain and inflammation."

A nurse is instructing a client diagnosed with coronary artery disease about care at home. The nurse determines that teaching is effective when the client states: SELECT ALL THAT APPLY. 1. "If I have chest pain, I should contact my physician immediately." 2. "I should carry my nitroglycerin in my front pants pocket so it is handy." 3. "If I have chest pain, I stop activity and place one nitroglycerin tablet under my tongue." 4. "I should always take three nitroglycerin tablets, 5 minutes apart." 5. "I plan to avoid being around people when they are smoking." 6. "I plan on walking on most days of the week for at least 30 minutes."

3. "If I have chest pain, I stop activity and place one nitroglycerin tablet under my tongue." 5. "I plan to avoid being around people when they are smoking." 6. "I plan on walking on most days of the week for at least 30 minutes."

The nurse is examining a 5-year-old boy who has diarrhea and fever. The caregiver states that the boy is normally active and healthy. On examination, the nurse hears a murmur at the LSB. The caregiver asks why the pediatrician has never said anything about it. The nurse explains: 1. "The pediatrician is not a cardiologist." 2. "Murmurs are difficult to detect, especially in children." 3. "The fever increased the intensity of the murmur." 4. "We need to refer the child to an interventional cardiologist."

3. "The fever increased the intensity of the murmur."

Which patient could require feeding by gavage? 1. A patient with KD in the acute phase. 2. A toddler with repair of transposition of the great vessels. 3. An infant with CHF. 4. A school-ager with RF and chorea.

3. An infant with CHF.

Blood for cardiac enzymes and serum laboratory tests are drawn on a diabetic client admitted to an emer gency department (ED) 5 hours after beginning to ex perience chest pressure. A nurse reviews the follow ing laboratory results. Which serum laboratory findings should the nurse report to a primary health care provider (HCP) immediately due to the possibil ity that the client may be experiencing a myocardial infarction (MI)? SELECT ALL THAT APPLY. 1. SCr 2. PT/INR 3. CK 4. CK-MB 5. Platelets 6. Troponin T

3. CK 4. CK-MB 6. Troponin T

. In which of the following CHDs would the nurse need to take upper and lower extremity BPs? 1. Transposition of the great vessels. 2. AS. 3. COA. 4. TOF.

3. COA.

A nurse is assessing a client diagnosed with an anterior-lateral myocardial infarction (MI). The nurse adds a nursing diagnosis to the client's plan of care of decreased cardiac output when which finding is noted on assessment? 1. One-sided weakness 2. Presence of an S4 heart sound 3. Crackles auscultated in bilateral lung bases 4. Vesicular breath sounds over lung lobes

3. Crackles auscultated in bilateral lung bases

CHDs are classified by which of the following? Select all that apply. 1. Cyanotic defect. 2. Acyanotic defect. 3. Defects with increased pulmonary blood flow. 4. Defects with decreased pulmonary blood flow. 5. Mixed defects. 6. Obstructive defects. 7. Pansystolic murmurs.

3. Defects with increased pulmonary blood flow. 4. Defects with decreased pulmonary blood flow. 5. Mixed defects. 6. Obstructive defects.

A 2-month-old is being treated with furosemide for CHF. Which of the following plans would also be appropriate in helping to control the CHF? 1. Promoting fluid restriction. 2. Feeding a low-salt formula. 3. Feeding in semi-Fowler position. 4. Encouraging breast milk.

3. Feeding in semi-Fowler position.

A 5-month-old has been diagnosed with an ASD. The flow of blood through the heart with this type of defect is: 1. Right to left. 2. Equal between the two chambers. 3. Left to right. 4. Bypassing the defect

3. Left to right.

. Heart transplant may be indicated for a child with which of the following symptoms? 1. Severe heart failure and PDA. 2. Severe heart failure and VSD. 3. Severe heart failure and hypoplastic left heart syndrome. 4. Severe heart failure and PS.

3. Severe heart failure and hypoplastic left heart syndrome.

During play, a toddler with a history of TOF might assume which of the following positions? 1. Sitting. 2. Supine. 3. Squatting. 4. Left lateral recumbent.

3. Squatting.

A male client confides to a clinic nurse that he is no longer dyspneic after receiving his new St. Jude's heart valve. He wants to have a vasectomy so that he can enjoy sexual intercourse again without the fear of his wife becoming pregnant. What is the nurse's best response? 1. "That's probably a good idea. The life expectancy after heart valve replacement is 10 to 15 years." 2. "You seem relieved that the heart valve replacement was successful and that you can enjoy a normal life again." 3. "If you have cardiac symptoms such as dyspnea during sexual intercourse, you can take a nitroglycerin tablet before sexual activity to prevent symptoms." 4. "Be sure to inform the physician that you have an artificial heart valve so you are given antibiotics as a preventive measure before the procedure."

4. "Be sure to inform the physician that you have an artificial heart valve so you are given antibiotics as a preventive measure before the procedure."

The school-aged female for whom you are caring has been diagnosed with valvular disease following RF. During patient teaching, you discuss the child's long-term prophylactic therapy with antibiotics for dental procedures, surgery, and childbirth. The parents indicate they understand when they say: 1. "She will need to take the antibiotics until she is 18 years old." 2. "She will need to take the antibiotics for 5 years after the last attack." 3. "She will need to take the antibiotics for 10 years after the last attack." 4. "She will need to take the antibiotics for the rest of her life."

4. "She will need to take the antibiotics for the rest of her life."

A nurse assesses a client who has just returned to a telemetry unit after having a coronary angiogram us ing the left femoral artery approach. The client's baseline blood pressure (BP) during the procedure was 130/72 mm Hg and the cardiac rhythm was a normal sinus throughout. Which assessment finding should indicate to the nurse that the client may be ex periencing a complication? 1. BP 144/78 mm Hg 2. Pedal pulses palpable at +1 3. Left groin soft with 1 cm ecchymotic area 4. Apical pulse 132 beats per minute (bpm) with an irregular-irregular rhythm

4. Apical pulse 132 beats per minute (bpm) with an irregular-irregular rhythm

The nurse is caring for a child who has undergone cardiac catheterization. During the recovery phase, the nurse notices the dressing is saturated with bright red blood and a 6-inch circle of blood on the crib sheet. The nurse's first action is to: 1. Call the interventional cardiologist. 2. Notify the cardiac catheterization laboratory that the child will be returning. 3. Apply a bulky pressure dressing over the present dressing. 4. Apply direct pressure 1 inch above the puncture site.

4. Apply direct pressure 1 inch above the puncture site.

A client admitted to a telemetry unit with a diagnosis of Prinzmetal's angina, has the following medications ordered. Upon interpretation of the client's electro cardiogram (ECG) rhythm, the nurse notes a prolonged PR interval of 0.32 second. Based on this information, which medication order should the nurse question administering to the client? 1. Isosorbide mononitrate (Imdur®) 20 mg oral daily upon awakening 2. Amlodipine (Norvasc®) 10 mg oral daily 3. Nitroglycerin (Nitrostat®) 0.4 mg sublingual prn for chest pain 4. Atenolol (Tenormin®) 50 mg oral daily

4. Atenolol (Tenormin®) 50 mg oral daily

A nurse collects the following assessment data on a client who has no known health problems: blood pressure (BP) 135/89 mm Hg; body mass index (BMI) 23; waist circumference 34 inches; serum creatinine 0.9 mg/dL; serum K 4.0 mEq/L; low-density lipopro tein (LDL) cholesterol 200 mg/dL; high-density lipoprotein (HDL) cholesterol 25 mg/dL; and triglyc erides 180 mg/dL. Which order from the client's health-care provider should the nurse anticipate? 1. 1,500-calorie regular diet. 2. No added salt, low saturated fat, low-potassium diet. 3. Hydrochlorothiazide (HydroDIURIL®) 25 mg twice daily. 4. Atorvastatin (Lipitor®) 20 mg daily

4. Atorvastatin (Lipitor®) 20 mg daily

A nurse, assessing a client hospitalized following a myocardial infarction (MI), obtains the following vital signs: blood pressure (BP) 78/38 mm Hg, heart rate (HR) 128, respiratory rate (RR) 32. For which life-threatening complication should the nurse care fully monitor the client? 1. Pulonary embolism 2. Cardiac tamponade 3. Cardiomyopathy 4. Cardiogenic shock

4. Cardiogenic shock

A nurse notes that a client, who experienced a myocardial infarction (MI) 3 days ago, seems unusually fatigued. Upon assessment, the nurse finds that the client is dyspneic with activity, has a heart rate (HR) of 110 beats per minute (bpm), and has generalized edema. Which action by the nurse is most appropriate? 1. Administer high-flow oxygen 2. Encourage the client to rest more 3. Continue to monitor the client's heart rhythm 4. Compare the client's admission weight with the client's current weight

4. Compare the client's admission weight with the client's current weight

A 2-month-old with TOF is seen in your clinic for a check-up. During the examination, the child develops severe respiratory distress and becomes cyanotic. The nurse's first action should be to: 1. Lay the child flat to promote hemostasis. 2. Lay the child flat with legs elevated to increase blood flow to the heart. 3. Sit the child on the parent's lap, with legs dangling, to promote venous pooling. 4. Hold the child in knee-chest position to decrease venous blood return.

4. Hold the child in knee-chest position to decrease venous blood return.

Gamma globulin is being given to a 1-year-old being treated for KD. Which of the following vaccines must be delayed for 11 months after the administration of gamma globulin? 1. Diphtheria, tetanus, and pertussis. 2. Hepatitis B. 3. Inactivated polio virus. 4. Measles, mumps, and rubella.

4. Measles, mumps, and rubella.

The nurse is caring for a toddler who has been hospitalized for 2 days with vomiting due to gastroenteritis. During morning assessment, she is sleeping and difficult to wake up. Assessment reveals vital signs of a regular HR of 220 beats per minute, respiratory rate of 30 per minute, BP of 84/52, and capillary refill of 3 seconds. Which dysrhythmia does the nurse suspect in this child? 1. Rapid pulmonary flutter. 2. Sinus bradycardia. 3. Rapid atrial fibrillation. 4. Supraventricular tachycardia.

4. Supraventricular tachycardia.

A client diagnosed with class II heart failure according to the New York Heart Association Func tional Classification has been taught about the initial treatment plan for this disease. A nurse determines that the client needs additional teaching if the client states that the treatment plan includes: 1. diuretics. 2. a low-sodium diet. 3. home oxygen therapy. 4. angiotensin-converting enzyme (ACE) inhibitors.

4. angiotensin-converting enzyme (ACE) inhibitors.

A nurse is caring for a client with a history of renal insufficiency who is scheduled for a cardiac catheterization. Which actions should the nurse take prior to the catheterization? (Select all that apply.) a. Assess for allergies to iodine. b. Administer intravenous fluids. c. Assess blood urea nitrogen (BUN) and creatinine results. d. Insert a Foley catheter. e. Administer a prophylactic antibiotic. f. Insert a central venous catheter.

ABC. a. Assess for allergies to iodine. b. Administer intravenous fluids. c. Assess blood urea nitrogen (BUN) and creatinine results.

A nurse who is beginning a shift on a cardiac step down unit receives shift report for four clients. In which order should the nurse assess the clients? Prioritize the nurse's actions by placing each client in order from most urgent (1) to least urgent (4). ______ A 56-year-old client who was admitted 1 day ago with chest pain receiving intravenous (IV) heparin and has a partial thromboplastin time (PTT) due back in 30 minutes ______ A 62-year-old client with end-stage cardiomyopathy, blood pressure (BP) of 78/50 mm Hg, 20 mL/hr urine output, and a "Do Not Resuscitate" order and whose family has just arrived ______ A 72-year-old client who was transferred 2 hours ago from the intensive care unit (ICU) following a coronary artery bypass graft and has new onset atrial fibrillation with rapid ventricular response ______ A 38-year-old postoperative client who had an aortic valve replacement 2 days ago, BP 114/72 mm Hg, heart rate (HR) 100 beats/min, respiratory rate (RR) 28 breaths/min, and temperature 101.2°F (38.4°C)

Answer: 3412

A nurse assesses clients on a cardiac unit. Which client should the nurse identify as being at greatest risk for the development of left-sided heart failure? a. A 36-year-old woman with aortic stenosis b. A 42-year-old man with pulmonary hypertension c. A 59-year-old woman who smokes cigarettes daily d. A 70-year-old man who had a cerebral vascular accident

a. A 36-year-old woman with aortic stenosis

A nurse assesses clients on a cardiac unit. Which clients should the nurse identify as at greatest risk for the development of acute pericarditis? (Select all that apply.) a. A 36-year-old woman with systemic lupus erythematosus (SLE) b. A 42-year-old man recovering from coronary artery bypass graft surgery c. A 59-year-old woman recovering from a hysterectomy d. An 80-year-old man with a bacterial infection of the respiratory tract e. An 88-year-old woman with a stage III sacral ulcer

a. A 36-year-old woman with systemic lupus erythematosus (SLE) b. A 42-year-old man recovering from coronary artery bypass graft surgery d. An 80-year-old man with a bacterial infection of the respiratory tract

A nursing student studying acute coronary syndromes learns that the pain of a myocardial infarction (MI) differs from stable angina in what ways? (Select all that apply.) a. Accompanied by shortness of breath b. Feelings of fear or anxiety c. Lasts less than 15 minutes d. No relief from taking nitroglycerin e. Pain occurs without known cause

a. Accompanied by shortness of breath b. Feelings of fear or anxiety d. No relief from taking nitroglycerin e. Pain occurs without known cause

A nursing student planning to teach clients about risk factors for coronary artery disease (CAD) would include which topics? (Select all that apply.) a. Advanced age b. Diabetes c. Ethnic background d. Medication use e. Smoking

a. Advanced age b. Diabetes c. Ethnic background e. Smoking

A nurse is interested in providing community education and screening on hypertension. In order to reach a priority population, to what target audience should the nurse provide this service? a. African-American churches b. Asian-American groceries c. High school sports camps d. Womens health clinics

a. African-American churches

A nurse is caring for a client who is intubated and has an intra-aortic balloon pump. The client is restless and agitated. What action should the nurse perform first for comfort? a. Allow family members to remain at the bedside. b. Ask the family if the client would like a fan in the room. c. Keep the television tuned to the clients favorite channel. d. Speak loudly to the client in case of hearing problems.

a. Allow family members to remain at the bedside.

A client has been bedridden for several days after major abdominal surgery. What action does the nurse delegate to the unlicensed assistive personnel (UAP) for deep vein thrombosis (DVT) prevention? (Select all that apply.) a. Apply compression stockings. b. Assist with ambulation. c. Encourage coughing and deep breathing. d. Offer fluids frequently. e. Teach leg exercises.

a. Apply compression stockings. b. Assist with ambulation. d. Offer fluids frequently.

A nurse prepares to discharge a client who has heart failure. Which questions should the nurse ask to ensure this clients safety prior to discharging home? (Select all that apply.) a. Are your bedroom and bathroom on the first floor? b. What social support do you have at home? c. Will you be able to afford your oxygen therapy? d. What spiritual beliefs may impact your recovery? e. Are you able to accurately weigh yourself at home?

a. Are your bedroom and bathroom on the first floor? b. What social support do you have at home? d. What spiritual beliefs may impact your recovery?

A nurse is working with a client who takes atorvastatin (Lipitor). The clients recent laboratory results include a blood urea nitrogen (BUN) of 33 mg/dL and creatinine of 2.8 mg/dL. What action by the nurse is best? a. Ask if the client eats grapefruit. b. Assess the client for dehydration. c. Facilitate admission to the hospital. d. Obtain a random urinalysis.

a. Ask if the client eats grapefruit.

A nurse is caring for a client with a nonhealing arterial ulcer. The physician has informed the client about possibly needing to amputate the clients leg. The client is crying and upset. What actions by the nurse are best? (Select all that apply.) a. Ask the client to describe his or her current emotions. b. Assess the client for support systems and family. c. Offer to stay with the client if he or she desires. d. Relate how smoking contributed to this situation. e. Tell the client that many people have amputations

a. Ask the client to describe his or her current emotions. b. Assess the client for support systems and family. c. Offer to stay with the client if he or she desires.

A nurse cares for a client who is on a cardiac monitor. The monitor displayed the rhythm shown below: Which action should the nurse take first? a. Assess airway, breathing, and level of consciousness. b. Administer an amiodarone bolus followed by a drip. c. Cardiovert the client with a biphasic defibrillator. d. Begin cardiopulmonary resuscitation (CPR).

a. Assess airway, breathing, and level of consciousness.

An older adult is on cardiac monitoring after a myocardial infarction. The client shows frequent dysrhythmias. What action by the nurse is most appropriate? a. Assess for any hemodynamic effects of the rhythm. b. Prepare to administer antidysrhythmic medication. c. Notify the provider or call the Rapid Response Team. d. Turn the alarms off on the cardiac monitor.

a. Assess for any hemodynamic effects of the rhythm.

While assessing a client on a cardiac unit, a nurse identifies the presence of an S3 gallop. Which action should the nurse take next? a. Assess for symptoms of left-sided heart failure. b. Document this as a normal finding. c. Call the health care provider immediately. d. Transfer the client to the intensive care unit.

a. Assess for symptoms of left-sided heart failure.

A client is on a dopamine infusion via a peripheral line. What action by the nurse takes priority for safety? a. Assess the IV site hourly. b. Monitor the pedal pulses. c. Monitor the clients vital signs. d. Obtain consent for a central line.

a. Assess the IV site hourly.

A nurse is caring for a client on IV infusion of heparin. What actions does this nurse include in the clients plan of care? (Select all that apply.) a. Assess the client for bleeding. b. Monitor the daily activated partial thromboplastin time (aPTT) results. c. Stop the IV for aPTT above baseline. d. Use an IV pump for the infusion. e. Weigh the client daily on the same scale.

a. Assess the client for bleeding. b. Monitor the daily activated partial thromboplastin time (aPTT) results. d. Use an IV pump for the infusion.

The nurse is assessing a client on admission to the hospital. The clients leg appears as shown below: What action by the nurse is best? a. Assess the clients ankle-brachial index. b. Elevate the clients leg above the heart. c. Obtain an ice pack to provide comfort. d. Prepare to teach about heparin sodium.

a. Assess the clients ankle-brachial index.

A client had an inferior wall myocardial infarction (MI). The nurse notes the clients cardiac rhythm as shown below: What action by the nurse is most important? a. Assess the clients blood pressure and level of consciousness. b. Call the health care provider or the Rapid Response Team. c. Obtain a permit for an emergency temporary pacemaker insertion. d. Prepare to administer antidysrhythmic medication.

a. Assess the clients blood pressure and level of consciousness.

A client with a history of heart failure and hypertension is in the clinic for a follow-up visit. The client is on lisinopril (Prinivil) and warfarin (Coumadin). The client reports new-onset cough. What action by the nurse is most appropriate? a. Assess the clients lung sounds and oxygenation. b. Instruct the client on another antihypertensive. c. Obtain a set of vital signs and document them. d. Remind the client that cough is a side effect of Prinivil.

a. Assess the clients lung sounds and oxygenation.

A nurse admits a client who is experiencing an exacerbation of heart failure. Which action should the nurse take first? a. Assess the clients respiratory status. b. Draw blood to assess the clients serum electrolytes. c. Administer intravenous furosemide (Lasix). d. Ask the client about current medications.

a. Assess the clients respiratory status.

A client has been diagnosed with a deep vein thrombosis and is to be discharged on warfarin (Coumadin). The client is adamant about refusing the drug because its dangerous. What action by the nurse is best? a. Assess the reason behind the clients fear. b. Remind the client about laboratory monitoring. c. Tell the client drugs are safer today than before. d. Warn the client about consequences of noncompliance.

a. Assess the reason behind the clients fear.

A nurse is caring for a client who had coronary artery bypass grafting yesterday. What actions does the nurse delegate to the unlicensed assistive personnel (UAP)? (Select all that apply.) a. Assist the client to the chair for meals and to the bathroom. b. Encourage the client to use the spirometer every 4 hours. c. Ensure the client wears TED hose or sequential compression devices. d. Have the client rate pain on a 0-to-10 scale and report to the nurse. e. Take and record a full set of vital signs per hospital protocol.

a. Assist the client to the chair for meals and to the bathroom. c. Ensure the client wears TED hose or sequential compression devices. e. Take and record a full set of vital signs per hospital protocol.

The nurse working in the emergency department knows that which factors are commonly related to aneurysm formation? (Select all that apply.) a. Atherosclerosis b. Down syndrome c. Frequent heartburn d. History of hypertension e. History of smoking

a. Atherosclerosis d. History of hypertension e. History of smoking

A nurse is teaching a client with heart failure who has been prescribed enalapril (Vasotec). Which statement should the nurse include in this clients teaching? a. Avoid using salt substitutes. b. Take your medication with food. c. Avoid using aspirin-containing products. d. Check your pulse daily.

a. Avoid using salt substitutes.

A nurse supervises an unlicensed assistive personnel (UAP) applying electrocardiographic monitoring. Which statement should the nurse provide to the UAP related to this procedure? a. Clean the skin and clip hairs if needed. b. Add gel to the electrodes prior to applying them. c. Place the electrodes on the posterior chest. d. Turn off oxygen prior to monitoring the client.

a. Clean the skin and clip hairs if needed.

A nurse assesses a client who is recovering from a myocardial infarction. The clients pulmonary artery pressure reading is 25/12 mm Hg. Which action should the nurse take first? a. Compare the results with previous pulmonary artery pressure readings. b. Increase the intravenous fluid rate because these readings are low. c. Immediately notify the health care provider of the elevated pressures. d. Document the finding in the clients chart as the only action.

a. Compare the results with previous pulmonary artery pressure readings.

A nurse is caring for a client with a nonhealing arterial lower leg ulcer. What action by the nurse is best? a. Consult with the Wound Ostomy Care Nurse. b. Give pain medication prior to dressing changes. c. Maintain sterile technique for dressing changes. d. Prepare the client for eventual amputation.

a. Consult with the Wound Ostomy Care Nurse.

A nursing student is caring for a client who had a myocardial infarction. The student is confused because the client states nothing is wrong and yet listens attentively while the student provides education on lifestyle changes and healthy menu choices. What response by the faculty member is best? a. Continue to educate the client on possible healthy changes. b. Emphasize complications that can occur with noncompliance. c. Tell the client that denial is normal and will soon go away. d. You need to make sure the client understands this illness.

a. Continue to educate the client on possible healthy changes.

A nurse is assessing a client with peripheral artery disease (PAD). The client states walking five blocks is possible without pain. What question asked next by the nurse will give the best information? a. Could you walk further than that a few months ago? b. Do you walk mostly uphill, downhill, or on flat surfaces? c. Have you ever considered swimming instead of walking? d. How much pain medication do you take each day?

a. Could you walk further than that a few months ago?

A nurse cares for a client with congestive heart failure who has a regular cardiac rhythm of 128 beats/min. For which physiologic alterations should the nurse assess? (Select all that apply.) a. Decrease in cardiac output b. Increase in cardiac output c. Decrease in blood pressure d. Increase in blood pressure e. Decrease in urine output f. Increase in urine output

a. Decrease in cardiac output d. Increase in blood pressure e. Decrease in urine output

A client is being discharged on warfarin (Coumadin) therapy. What discharge instructions is the nurse required to provide? (Select all that apply.) a. Dietary restrictions b. Driving restrictions c. Follow-up laboratory monitoring d. Possible drug-drug interactions e. Reason to take medication

a. Dietary restrictions c. Follow-up laboratory monitoring d. Possible drug-drug interactions e. Reason to take medication

A client is in the clinic a month after having a myocardial infarction. The client reports sleeping well since moving into the guest bedroom. What response by the nurse is best? a. Do you have any concerns about sexuality? b. Im glad to hear you are sleeping well now. c. Sleep near your spouse in case of emergency. d. Why would you move into the guest room?

a. Do you have any concerns about sexuality?

The nurse is caring for four hypertensive clients. Which druglaboratory value combination should the nurse report immediately to the health care provider? a. Furosemide (Lasix)/potassium: 2.1 mEq/L b. Hydrochlorothiazide (Hydrodiuril)/potassium: 4.2 mEq/L c. Spironolactone (Aldactone)/potassium: 5.1 mEq/L d. Torsemide (Demadex)/sodium: 142 mEq/L

a. Furosemide (Lasix)/potassium: 2.1 mEq/L

A nurse assesses a client who had a myocardial infarction and is hypotensive. Which additional assessment finding should the nurse expect? a. Heart rate of 120 beats/min b. Cool, clammy skin c. Oxygen saturation of 90% d. Respiratory rate of 8 breaths/min

a. Heart rate of 120 beats/min

A nurse evaluates laboratory results for a client with heart failure. Which results should the nurse expect? (Select all that apply.) a. Hematocrit: 32.8% b. Serum sodium: 130 mEq/L c. Serum potassium: 4.0 mEq/L d. Serum creatinine: 1.0 mg/dL e. Proteinuria f. Microalbuminuria

a. Hematocrit: 32.8% b. Serum sodium: 130 mEq/L e. Proteinuria f. Microalbuminuria

A client has peripheral arterial disease (PAD). What statement by the client indicates misunderstanding about self-management activities? a. I can use a heating pad on my legs if its set on low. b. I should not cross my legs when sitting or lying down. c. I will go out and buy some warm, heavy socks to wear. d. Its going to be really hard but I will stop smoking.

a. I can use a heating pad on my legs if its set on low.

An emergency room nurse obtains the health history of a client. Which statement by the client should alert the nurse to the occurrence of heart failure? a. I get short of breath when I climb stairs. b. I see halos floating around my head. c. I have trouble remembering things. d. I have lost weight over the past month.

a. I get short of breath when I climb stairs.

After teaching a client with congestive heart failure (CHF), the nurse assesses the clients understanding. Which client statements indicate a correct understanding of the teaching related to nutritional intake? (Select all that apply.) a. Ill read the nutritional labels on food items for salt content. b. I will drink at least 3 liters of water each day. c. Using salt in moderation will reduce the workload of my heart. d. I will eat oatmeal for breakfast instead of ham and eggs. e. Substituting fresh vegetables for canned ones will lower my salt intake.

a. Ill read the nutritional labels on food items for salt content. d. I will eat oatmeal for breakfast instead of ham and eggs. e. Substituting fresh vegetables for canned ones will lower my salt intake.

Which of the following statements about myocardial infarction pain is incorrect? a. It is relieved by rest and inactivity. b. It is substernal in location. c. It is sudden in onset and prolonged in duration. d. It is viselike and radiates to the shoulders and arms.

a. It is relieved by rest and inactivity.

A nurse assesses a client who has aortic regurgitation. In which location in the illustration shown below should the nurse auscultate to best hear a cardiac murmur related to aortic regurgitation? a. Location A b. Location B c. Location C d. Location D

a. Location A

A nurse prepares to discharge a client with cardiac dysrhythmia who is prescribed home health care services. Which priority information should be communicated to the home health nurse upon discharge? a. Medication reconciliation b. Immunization history c. Religious beliefs d. Nutrition preferences

a. Medication reconciliation

A nurse assesses a client with tachycardia. Which clinical manifestation requires immediate intervention by the nurse? a. Mid-sternal chest pain b. Increased urine output c. Mild orthostatic hypotension d. P wave touching the T wave

a. Mid-sternal chest pain

A nurse teaches a client who experiences occasional premature atrial contractions (PACs) accompanied by palpitations that resolve spontaneously without treatment. Which statement should the nurse include in this clients teaching? a. Minimize or abstain from caffeine. b. Lie on your side until the attack subsides. c. Use your oxygen when you experience PACs. d. Take amiodarone (Cordarone) daily to prevent PACs.

a. Minimize or abstain from caffeine.

A client is taking warfarin (Coumadin) and asks the nurse if taking St. Johns wort is acceptable. What response by the nurse is best? a. No, it may interfere with the warfarin. b. There isnt any information about that. c. Why would you want to take that? d. Yes, it is a good supplement for you.

a. No, it may interfere with the warfarin.

A nurse is teaching a larger female client about alcohol intake and how it affects hypertension. The client asks if drinking two beers a night is an acceptable intake. What answer by the nurse is best? a. No, women should only have one beer a day as a general rule. b. No, you should not drink any alcohol with hypertension. c. Yes, since you are larger, you can have more alcohol. d. Yes, two beers per day is an acceptable amount of alcohol.

a. No, women should only have one beer a day as a general rule.

A nurse is assessing a client with left-sided heart failure. For which clinical manifestations should the nurse assess? (Select all that apply.) a. Pulmonary crackles b. Confusion, restlessness c. Pulmonary hypertension d. Dependent edema e. Cough that worsens at night

a. Pulmonary crackles b. Confusion, restlessness e. Cough that worsens at night

A nurse collaborates with an unlicensed assistive personnel (UAP) to provide care for a client with congestive heart failure. Which instructions should the nurse provide to the UAP when delegating care for this client? (Select all that apply.) a. Reposition the client every 2 hours. b. Teach the client to perform deep-breathing exercises. c. Accurately record intake and output. d. Use the same scale to weigh the client each morning. e. Place the client on oxygen if the client becomes short of breath

a. Reposition the client every 2 hours. c. Accurately record intake and output. d. Use the same scale to weigh the client each morning.

While examining Mr. Fontana's eyes during the admission assessment, the nurse notes conjunctival hemorrhages with pale centers caused by emboli in the nerve fiber of the eye. These are known as: a. Roth's spots. b. Osler's nodes. c. Janeway's lesions. d. Heberden's nodes.

a. Roth's spots.

A nurse assesses a client who is recovering from a heart transplant. Which assessment findings should alert the nurse to the possibility of heart transplant rejection? (Select all that apply.) a. Shortness of breath b. Abdominal bloating c. New-onset bradycardia d. Increased ejection fraction e. Hypertension

a. Shortness of breath b. Abdominal bloating c. New-onset bradycardia

A nurse is teaching a client with premature ectopic beats. Which education should the nurse include in this clients teaching? (Select all that apply.) a. Smoking cessation b. Stress reduction and management c. Avoiding vagal stimulation d. Adverse effects of medications e. Foods high in potassium

a. Smoking cessation b. Stress reduction and management d. Adverse effects of medications

A nurse cares for a client with infective endocarditis. Which infection control precautions should the nurse use? a. Standard Precautions b. Bleeding precautions c. Reverse isolation d. Contact isolation

a. Standard Precautions

A nurse prepares to discharge a client who has heart failure. Based on the Heart Failure Core Measure Set, which actions should the nurse complete prior to discharging this client? (Select all that apply.) a. Teach the client about dietary restrictions. b. Ensure the client is prescribed an angiotensin-converting enzyme (ACE) inhibitor. c. Encourage the client to take a baby aspirin each day. d. Confirm that an echocardiogram has been completed. e. Consult a social worker for additional resource

a. Teach the client about dietary restrictions. b. Ensure the client is prescribed an angiotensin-converting enzyme (ACE) inhibitor. d. Confirm that an echocardiogram has been completed.

A nurse teaches a client with a new permanent pacemaker. Which instructions should the nurse include in this clients teaching? (Select all that apply.) a. Until your incision is healed, do not submerge your pacemaker. Only take showers. b. Report any pulse rates lower than your pacemaker settings. c. If you feel weak, apply pressure over your generator. d. Have your pacemaker turned off before having magnetic resonance imaging (MRI). e. Do not lift your left arm above the level of your shoulder for 8 weeks

a. Until your incision is healed, do not submerge your pacemaker. Only take showers. b. Report any pulse rates lower than your pacemaker settings. e. Do not lift your left arm above the level of your shoulder for 8 weeks

A nurse cares for a client with right-sided heart failure. The client asks, Why do I need to weigh myselfevery day? How should the nurse respond? a. Weight is the best indication that you are gaining or losing fluid. b. Daily weights will help us make sure that youre eating properly. c. The hospital requires that all inpatients be weighed daily. d. You need to lose weight to decrease the incidence of heart failure.

a. Weight is the best indication that you are gaining or losing fluid.

A nurse cares for an older adult client with heart failure. The client states, I dont know what to do. I dont want to be a burden to my daughter, but I cant do it alone. Maybe I should die. How should the nurse respond? a. Would you like to talk more about this? b. You are lucky to have such a devoted daughter. c. It is normal to feel as though you are a burden. d. Would you like to meet with the chaplain?

a. Would you like to talk more about this?

The causative microorganism for rheumatic endocarditis can be accurately identified only by: a. a throat culture. b. an echocardiogram. c. roentgenography. d. serum analysis.

a. a throat culture.

The most reliable sign of cardiac arrest is: a. absence of a pulse. b. cessation of respirations. c. dilation of the pupils. d. inaudible heart sounds.

a. absence of a pulse.

Fred, a 43-year-old construction worker, has a history of hypertension. He smokes two packs of cigarettes a day, is nervous about the possibility of being unemployed, and has difficulty coping with stress. His current concern is calf pain during minimal exercise, which decreases with rest. 1. The nurse assesses Fred's symptoms as being associated with peripheral arterial occlusive disease. The nursing diagnosis is probably: a. alteration in tissue perfusion related to compromised circulation. b. dysfunctional use of extremities related to muscle spasms. c. impaired mobility related to stress associated with pain. d. impairment in muscle use associated with pain on exertion

a. alteration in tissue perfusion related to compromised circulation.

The classic ECG changes that occur with an MI include all of the following except: a. an absent P wave. b. an abnormal Q wave. c. T-wave inversion. d. ST-segment elevation.

a. an absent P wave.

A primary classification of medications used in the treatment of systolic heart failure is: a. angiotensin-converting enzyme inhibitors. b. beta-blockers. c. diuretics. d. calcium-channel blockers.

a. angiotensin-converting enzyme inhibitors.

The coronary arteries arise from the: a. aorta near the origin of the left ventricle. b. pulmonary artery at the apex of the right ventricle. c. pulmonary vein near the left atrium. d. superior vena cava at the origin of the right atrium.

a. aorta near the origin of the left ventricle.

The presence of a water-hammer pulse (quick, sharp strokes that suddenly collapse) is diagnostic for: a. aortic regurgitation. b. mitral insufficiency. c. tricuspid insufficiency. d. tricuspid stenosis.

a. aortic regurgitation.

The scientific rationale supporting the administration of beta-adrenergic blockers is the drugs' ability to: a. block sympathetic impulses to the heart. b. elevate blood pressure. c. increase myocardial contractility. d. induce bradycardia.

a. block sympathetic impulses to the heart.

Classic signs of cardiogenic shock include all of the following except: a. bradycardia. b. cerebral hypoxia. c. hypotension. d. oliguria.

a. bradycardia.

A complication after cardiac surgery that is associated with an alteration in preload is: a. cardiac tamponade. b. elevated central venous pressure. c. hypertension. d. hypothermia.

a. cardiac tamponade.

A serious consequence of pericarditis is: a. cardiac tamponade. b. decreased venous pressure. c. hypertension. d. left ventricular hypertrophy

a. cardiac tamponade.

The nurse needs to look for symptoms associated with one of the major causes of sudden death during the first 48 hours, which is: a. cardiogenic shock. b. pulmonary edema. c. pulmonary embolism. d. ventricular rupture

a. cardiogenic shock.

The multilumen pulmonary artery catheter allows the nurse to measure hemodynamic pressures at various points in the heart. When the tip enters the small branches of the pulmonary artery, the nurse can assess all of the following measurements except: a. central venous pressure (CVP). b. pulmonary artery capillary pressure (PACP). c. pulmonary artery obstructive pressure (PAOP). d. pulmonary artery wedge pressure (PAWP).

a. central venous pressure (CVP).

Saturated fats are strongly implicated in the causation of atherosclerosis. Saturated fats include all of the following except: a. corn oil. b. eggs and milk. c. meat and butter. d. solid vegetable oil.

a. corn oil.

The pain of angina pectoris is produced primarily by: a. coronary vasoconstriction. b. movement of thromboemboli. c. myocardial ischemia. d. the presence of atheromas.

a. coronary vasoconstriction.

Postoperative nursing management for vein ligation and stripping include all of the following except: a. dangling the legs over the side of the bed for 10 minutes every 4 hours for the first 24 hours. b. elevating the foot of the bed to promote venous blood return. c. maintaining elastic compression of the leg continuously for about 1 week. d. starting the patient ambulating 24 to 48 hours after surgery.

a. dangling the legs over the side of the bed for 10 minutes every 4 hours for the first 24 hours.

The primary underlying disorder of pulmonary edema is: a. decreased left ventricular pumping. b. decreased right ventricular elasticity. c. increased left atrial contractility. d. increased right atrial resistance.

a. decreased left ventricular pumping.

Calcium channel blockers act by: a. decreasing SA node automaticity. b. increasing AV node conduction. c. increasing the heart rate. d. creating a positive inotropic effect

a. decreasing SA node automaticity.

The most commonly occurring cardiomyopathy is: a. dilated. b. hypertrophic. c. idiopathic. d. restrictive.

a. dilated.

2. The sequence of pathophysiologic events is triggered by: a. elevated left ventricular end-diastolic pressure. b. elevated pulmonary venous pressure. c. increased hydrostatic pressure. d. impaired lymphatic drainage.

a. elevated left ventricular end-diastolic pressure.

If the sphygmomanometer cuff is too small for the patient, the blood pressure reading will probably be: a. falsely elevated. b. falsely decreased. c. an accurate reading. d. significantly different with each reading.

a. falsely elevated.

Mrs. Russell is a 46-year-old Caucasian who developed symptoms of acute pericarditis secondary to a viral infection. Diagnosis was based on the characteristic sign of a friction rub and pain over the pericardium. 1. On the basis of knowledge of pericardial pain, the nurse suggests the following body position to relieve the pain symptoms: a. flat in bed with feet slightly higher than the head. b. Fowler's. c. right side-lying. d. semi-Fowler's.

a. flat in bed with feet slightly higher than the head.

Severe aortic stenotic disease is consistent with all of the following except: a. increased cardiac output. b. left ventricular hypertrophy. c. pulmonary edema. d. right-sided heart failure

a. increased cardiac output.

Mr. Lillis, a 46-year-old bricklayer, is brought to the emergency department by ambulance with a suspected diagnosis of myocardial infarction. He appears ashen, is diaphoretic and tachycardiac, and has severe chest pain. The nursing diagnosis is decreased cardiac output, related to decreased myocardial tissue perfusion. 1. The nurse knows that the most critical time period for his diagnosis is: a. the first hour after symptoms begin. b. within 24 hours after the onset of symptoms. c. within the first 48 hours after the attack. d. between the third and fifth day after the attack

a. the first hour after symptoms begin.

The nurse who is caring for a patient with pericarditis understands that there is inflammation involving the: a. thin fibrous sac encasing the heart. b. inner lining of the heart and valves. c. heart's muscle fibers. d. exterior layer of the heart.

a. thin fibrous sac encasing the heart.

The nurse is asked to determine ABI. The right posterior tibial reading is 75 mm Hg and the brachial systolic pressure is 150 mm Hg. The ABI would be: a. 0.25. b. 0.50. c. 0.65. d. 0.80.

b. 0.50.

In the United States, about 1 million people will have an acute myocardial infarction each year. Of these 1 million, what percentage will die? a. 10% to 15% b. 25% c. 30% to 40% d. 60%

b. 25%

The American diet is known to be high in fat. The amount of calories typically supplied by fat in most diets is ________ of the total caloric intake. a. 20% b. 35% c. 60% d. 80%

b. 35%

Brain damage occurs with cessation of circulation after an approximate interval of: a. 2 minutes. b. 4 minutes. c. 6 minutes. d. 8 minutes.

b. 4 minutes.

A nurse is assessing clients on a medical-surgical unit. Which client should the nurse identify as being at greatest risk for atrial fibrillation? a. A 45-year-old who takes an aspirin daily b. A 50-year-old who is post coronary artery bypass graft surgery c. A 78-year-old who had a carotid endarterectomy d. An 80-year-old with chronic obstructive pulmonary disease

b. A 50-year-old who is post coronary artery bypass graft surgery

A client is in the hospital after suffering a myocardial infarction and has bathroom privileges. The nurse assists the client to the bathroom and notes the clients O2 saturation to be 95%, pulse 88 beats/min, and respiratory rate 16 breaths/min after returning to bed. What action by the nurse is best? a. Administer oxygen at 2 L/min. b. Allow continued bathroom privileges. c. Obtain a bedside commode. d. Suggest the client use a bedpan.

b. Allow continued bathroom privileges.

A client has a deep vein thrombosis (DVT). What comfort measure does the nurse delegate to the unlicensed assistive personnel (UAP)? a. Ambulate the client. b. Apply a warm moist pack. c. Massage the clients leg. d. Provide an ice pack

b. Apply a warm moist pack.

What nonpharmacologic comfort measures should the nurse include in the plan of care for a client with severe varicose veins? (Select all that apply.) a. Administering mild analgesics for pain b. Applying elastic compression stockings c. Elevating the legs when sitting or lying d. Reminding the client to do leg exercises e. Teaching the client about surgical options

b. Applying elastic compression stockings c. Elevating the legs when sitting or lying d. Reminding the client to do leg exercises

A nurse assesses a client who has a history of heart failure. Which question should the nurse ask to assess the extent of the clients heart failure? a. Do you have trouble breathing or chest pain? b. Are you able to walk upstairs without fatigue? c. Do you awake with breathlessness during the night? d. Do you have new-onset heaviness in your legs?

b. Are you able to walk upstairs without fatigue?

A client is 4 hours postoperative after a femoropopliteal bypass. The client reports throbbing leg pain on the affected side, rated as 7/10. What action by the nurse takes priority? a. Administer pain medication as ordered. b. Assess distal pulses and skin color. c. Document the findings in the clients chart. d. Notify the surgeon immediately.

b. Assess distal pulses and skin color.

A client with a known abdominal aortic aneurysm reports dizziness and severe abdominal pain. The nurse assesses the clients blood pressure at 82/40 mm Hg. What actions by the nurse are most important? (Select all that apply.) a. Administer pain medication. b. Assess distal pulses every 10 minutes. c. Have the client sign a surgical consent. d. Notify the Rapid Response Team. e. Take vital signs every 10 minutes.

b. Assess distal pulses every 10 minutes. d. Notify the Rapid Response Team. e. Take vital signs every 10 minutes.

A client has intra-arterial blood pressure monitoring after a myocardial infarction. The nurse notes the clients heart rate has increased from 88 to 110 beats/min, and the blood pressure dropped from 120/82 to 100/60 mm Hg. What action by the nurse is most appropriate? a. Allow the client to rest quietly. b. Assess the client for bleeding. c. Document the findings in the chart. d. Medicate the client for pain.

b. Assess the client for bleeding.

A nurse cares for a client with an intravenous temporary pacemaker for bradycardia. The nurse observes the presence of a pacing spike but no QRS complex on the clients electrocardiogram. Which action should the nurse take next? a. Administer intravenous diltiazem (Cardizem). b. Assess vital signs and level of consciousness. c. Administer sublingual nitroglycerin. d. Assess capillary refill and temperature.

b. Assess vital signs and level of consciousness.

A client has hypertension and high risk factors for cardiovascular disease. The client is overwhelmed with the recommended lifestyle changes. What action by the nurse is best? a. Assess the clients support system. b. Assist in finding one change the client can control. c. Determine what stressors the client faces in daily life. d. Inquire about delegating some of the clients obligations.

b. Assist in finding one change the client can control.

A client is 1 day postoperative after a coronary artery bypass graft. What nonpharmacologic comfort measures does the nurse include when caring for this client? (Select all that apply.) a. Administer pain medication before ambulating. b. Assist the client into a position of comfort in bed. c. Encourage high-protein diet selections. d. Provide complementary therapies such as music. e. Remind the client to splint the incision when coughing.

b. Assist the client into a position of comfort in bed. d. Provide complementary therapies such as music. e. Remind the client to splint the incision when coughing.

A nurse assesses a client who has mitral valve regurgitation. For which cardiac dysrhythmia should the nurse assess? a. Preventricular contractions b. Atrial fibrillation c. Symptomatic bradycardia d. Sinus tachycardia

b. Atrial fibrillation

A nurse teaches a client recovering from a heart transplant who is prescribed cyclosporine (Sandimmune). Which statement should the nurse include in this clients discharge teaching? a. Use a soft-bristled toothbrush and avoid flossing. b. Avoid large crowds and people who are sick. c. Change positions slowly to avoid hypotension. d. Check your heart rate before taking the medication.

b. Avoid large crowds and people who are sick.

A nurse cares for a client who has a heart rate averaging 56 beats/min with no adverse symptoms. Which activity modification should the nurse suggest to avoid further slowing of the heart rate? a. Make certain that your bath water is warm. b. Avoid straining while having a bowel movement. c. Limit your intake of caffeinated drinks to one a day. d. Avoid strenuous exercise such as running.

b. Avoid straining while having a bowel movement.

The nurse is evaluating a 3-day diet history with a client who has an elevated lipid panel. What meal selection indicates the client is managing this condition well with diet? a. A 4-ounce steak, French fries, iceberg lettuce b. Baked chicken breast, broccoli, tomatoes c. Fried catfish, cornbread, peas d. Spaghetti with meat sauce, garlic bread

b. Baked chicken breast, broccoli, tomatoes

A nurse cares for a client recovering from prosthetic valve replacement surgery. The client asks, Why will I need to take anticoagulants for the rest of my life? How should the nurse respond? a. The prosthetic valve places you at greater risk for a heart attack. b. Blood clots form more easily in artificial replacement valves. c. The vein taken from your leg reduces circulation in the leg. d. The surgery left a lot of small clots in your heart and lungs.

b. Blood clots form more easily in artificial replacement valves.

The nurse knows that this angiotensin-converting enzyme inhibitor ordered by the physician has a rapid onset of action within 15 minutes. a. Altace b. Capoten c. Lotensin d. Vasotec

b. Capoten

A nurse is caring for four clients. Which one should the nurse see first? a. Client who needs a beta blocker, and has a blood pressure of 92/58 mm Hg b. Client who had a first dose of captopril (Capoten) and needs to use the bathroom c. Hypertensive client with a blood pressure of 188/92 mm Hg d. Client who needs pain medication prior to a dressing change of a surgical wound

b. Client who had a first dose of captopril (Capoten) and needs to use the bathroom

A nurse is in charge of the coronary intensive care unit. Which client should the nurse see first? a. Client on a nitroglycerin infusion at 5 mcg/min, not titrated in the last 4 hours b. Client who is 1 day post coronary artery bypass graft, blood pressure 180/100 mm Hg c. Client who is 1 day post percutaneous coronary intervention, going home this morning d. Client who is 2 days post coronary artery bypass graft, became dizzy this a.m. while walking

b. Client who is 1 day post coronary artery bypass graft, blood pressure 180/100 mm Hg

A nurse is caring for four clients. Which client should the nurse assess first? a. Client with an acute myocardial infarction, pulse 102 beats/min b. Client who is 1 hour post angioplasty, has tongue swelling and anxiety c. Client who is post coronary artery bypass, chest tube drained 100 mL/hr d. Client who is post coronary artery bypass, potassium 4.2 mEq/L

b. Client who is 1 hour post angioplasty, has tongue swelling and anxiety

An immunosuppressant that allowed heart transplantation to become a therapeutic option for end-stage heart disease is: a. Procardia. b. Cyclosporine. c. Calan. d. Vancocin.

b. Cyclosporine.

A client had a percutaneous transluminal coronary angioplasty for peripheral arterial disease. What assessment finding by the nurse indicates a priority outcome for this client has been met? a. Pain rated as 2/10 after medication b. Distal pulse on affected extremity 2+/4+ c. Remains on bedrest as directed d. Verbalizes understanding of procedure

b. Distal pulse on affected extremity 2+/4+

A nurse auscultated heart tones on an older adult client. Which action should the nurse take based on heart tones heard? (Click the media button to hear the audio clip.) a. Administer a diuretic. b. Document the finding. c. Decrease the IV flow rate. d. Evaluate the clients medications.

b. Document the finding.

A nurse assesses a client with mitral valve stenosis. What clinical manifestation should alert the nurse to the possibility that the clients stenosis has progressed? a. Oxygen saturation of 92% b. Dyspnea on exertion c. Muted systolic murmur d. Upper extremity weakness

b. Dyspnea on exertion

A client has hemodynamic monitoring after a myocardial infarction. What safety precaution does the nurse implement for this client? a. Document pulmonary artery wedge pressure (PAWP) readings and assess their trends. b. Ensure the balloon does not remain wedged. c. Keep the client on strict NPO status. d. Maintain the client in a semi-Fowlers position

b. Ensure the balloon does not remain wedged.

a. Administer oxygen via non-rebreather mask. b. Ensure the client has a patent airway. c. Prepare to assist with suturing the artery. d. Start two large-bore IVs with normal saline

b. Ensure the client has a patent airway.

A home health care nurse is visiting an older client who lives alone after being discharged from the hospital after a coronary artery bypass graft. What finding in the home most causes the nurse to consider additional referrals? a. Dirty carpets in need of vacuuming b. Expired food in the refrigerator c. Old medications in the kitchen d. Several cats present in the home

b. Expired food in the refrigerator

A nurse assesses a client with pericarditis. Which assessment finding should the nurse expect to find? a. Heart rate that speeds up and slows down b. Friction rub at the left lower sternal border c. Presence of a regular gallop rhythm d. Coarse crackles in bilateral lung bases

b. Friction rub at the left lower sternal border

. A nurse teaches a client with heart failure about energy conservation. Which statement should the nurse include in this clients teaching? a. Walk until you become short of breath, and then walk back home. b. Gather everything you need for a chore before you begin. c. Pull rather than push or carry items heavier than 5 pounds. d. Take a walk after dinner every day to build up your strength.

b. Gather everything you need for a chore before you begin.

A client has presented to the emergency department with an acute myocardial infarction (MI). What action by the nurse is best to meet The Joint Commissions Core Measures outcomes? a. Obtain an electrocardiogram (ECG) now and in the morning. b. Give the client an aspirin. c. Notify the Rapid Response Team. d. Prepare to administer thrombolytics.

b. Give the client an aspirin.

After teaching a client who has an implantable cardioverter-defibrillator (ICD), a nurse assesses the clients understanding. Which statement by the client indicates a correct understanding of the teaching? a. I should wear a snug-fitting shirt over the ICD. b. I will avoid sources of strong electromagnetic fields. c. I should participate in a strenuous exercise program. d. Now I can discontinue my antidysrhythmic medication.

b. I will avoid sources of strong electromagnetic fields.

After teaching a client who is being discharged home after mitral valve replacement surgery, the nurse assesses the clients understanding. Which client statement indicates a need for additional teaching? a. Ill be able to carry heavy loads after 6 months of rest. b. I will have my teeth cleaned by my dentist in 2 weeks. c. I must avoid eating foods high in vitamin K, like spinach. d. I must use an electric razor instead of a straight razor to shave

b. I will have my teeth cleaned by my dentist in 2 weeks.

On assessment, the nurse knows that a patient who reports no symptoms of heart failure at rest but is symptomatic with increased physical activity would have a heart failure classification of: a. I. b. II. c. III. d. IV.

b. II.

Which of the following medications would not be used to treat pericarditis because it can decrease blood flow? a. Colchicine b. Indocin c. Motrin d. Prednisone

b. Indocin

The nurse is caring for a client on the medical-surgical unit who suddenly becomes unresponsive and has no pulse. The cardiac monitor shows the rhythm below: After calling for assistance and a defibrillator, which action should the nurse take next? a. Perform a pericardial thump. b. Initiate cardiopulmonary resuscitation (CPR). c. Start an 18-gauge intravenous line. d. Ask the clients family about code status.

b. Initiate cardiopulmonary resuscitation (CPR).

A nurse cares for a client who has an 80% blockage of the right coronary artery (RCA) and is scheduled for bypass surgery. Which intervention should the nurse be prepared to implement while this client waits for surgery? a. Administration of IV furosemide (Lasix) b. Initiation of an external pacemaker c. Assistance with endotracheal intubation d. Placement of central venous access

b. Initiation of an external pacemaker

After administering newly prescribed captopril (Capoten) to a client with heart failure, the nurse implements interventions to decrease complications. Which priority intervention should the nurse implement for this client? a. Provide food to decrease nausea and aid in absorption. b. Instruct the client to ask for assistance when rising from bed. c. Collaborate with unlicensed assistive personnel to bathe the client. d. Monitor potassium levels and check for symptoms of hypokalemia

b. Instruct the client to ask for assistance when rising from bed.

A client has been diagnosed with hypertension but does not take the antihypertensive medications because of a lack of symptoms. What response by the nurse is best? a. Do you have trouble affording your medications? b. Most people with hypertension do not have symptoms. c. You are lucky; most people get severe morning headaches. d. You need to take your medicine or you will get kidney failure.

b. Most people with hypertension do not have symptoms.

A nurse assesses a client admitted to the cardiac unit. Which statement by the client alerts the nurse to the possibility of right-sided heart failure? a. I sleep with four pillows at night. b. My shoes fit really tight lately. c. I wake up coughing every night. d. I have trouble catching my breath.

b. My shoes fit really tight lately.

A nurse obtains the health history of a client who is newly admitted to the medical unit. Which statement by the client should alert the nurse to the presence of edema? a. I wake up to go to the bathroom at night. b. My shoes fit tighter by the end of the day. c. I seem to be feeling more anxious lately. d. I drink at least eight glasses of water a day.

b. My shoes fit tighter by the end of the day.

A nurse assesses a client who is diagnosed with infective endocarditis. Which assessment findings should the nurse expect? (Select all that apply.) a. Weight gain b. Night sweats c. Cardiac murmur d. Abdominal bloating e. Oslers nodes

b. Night sweats c. Cardiac murmur e. Oslers nodes

A client is receiving an infusion of alteplase (Activase) for an intra-arterial clot. The client begins to mumble and is disoriented. What action by the nurse takes priority? a. Assess the clients neurologic status. b. Notify the Rapid Response Team. c. Prepare to administer vitamin K. d. Turn down the infusion rate.

b. Notify the Rapid Response Team.

A nurse cares for a client who is prescribed magnetic resonance imaging (MRI) of the heart. The clients health history includes a previous myocardial infarction and pacemaker implantation. Which action should the nurse take? a. Schedule an electrocardiogram just before the MRI. b. Notify the health care provider before scheduling the MRI. c. Call the physician and request a laboratory draw for cardiac enzymes. d. Instruct the client to increase fluid intake the day before the MRI.

b. Notify the health care provider before scheduling the MRI.

The nurse is caring for a client with a chest tube after a coronary artery bypass graft. The drainage slows significantly. What action by the nurse is most important? a. Increase the setting on the suction. b. Notify the provider immediately. c. Re-position the chest tube. d. Take the tubing apart to assess for clots.

b. Notify the provider immediately.

A nurse wants to provide community service that helps meet the goals of Healthy People 2020 (HP2020) related to cardiovascular disease and stroke. What activity would best meet this goal? a. Teach high school students heart-healthy living. b. Participate in blood pressure screenings at the mall. c. Provide pamphlets on heart disease at the grocery store. d. Set up an Ask the nurse booth at the pet store.

b. Participate in blood pressure screenings at the mall.

The total time for ventricular depolarization and repolarization is represented on an electrocardiogram (ECG) reading as the: a. QRS complex. b. QT interval. c. ST segment. d. TP interval.

b. QT interval.

After assessing a client who is receiving an amiodarone intravenous infusion for unstable ventricular tachycardia, the nurse documents the findings and compares these with the previous assessment findings: Vital Signs Nursing Assessment Time: 0800 Temperature: 98 F Heart rate: 68 beats/min Blood pressure: 135/60 mm Hg Respiratory rate: 14 breaths/min Oxygen saturation: 96% Oxygen therapy: 2 L nasal cannula Time: 1000 Temperature: 98.2 F Heart rate: 50 beats/min Blood pressure: 132/57 mm Hg Respiratory rate: 16 breaths/min Oxygen saturation: 95% Oxygen therapy: 2 L nasal cannula Time: 0800 Client alert and oriented. Cardiac rhythm: normal sinus rhythm. Skin: warm, dry, and appropriate for race. Respirations equal and unlabored. Client denies shortness of breath and chest pain. Time: 1000 Client alert and oriented. Cardiac rhythm: sinus bradycardia. Skin: warm, dry, and appropriate for race. Respirations equal and unlabored. Client denies shortness of breath and chest pain. Client voids 420 mL of clear yellow urine. Based on the assessments, which action should the nurse take? a. Stop the infusion and flush the IV. b. Slow the amiodarone infusion rate. c. Administer IV normal saline. d. Ask the client to cough and deep breathe.

b. Slow the amiodarone infusion rate.

A nurse assesses a client with atrial fibrillation. Which manifestation should alert the nurse to the possibility of a serious complication from this condition? a. Sinus tachycardia b. Speech alterations c. Fatigue d. Dyspnea with activity

b. Speech alterations

A client with coronary artery disease (CAD) asks the nurse about taking fish oil supplements. What response by the nurse is best? a. Fish oil is contraindicated with most drugs for CAD. b. The best source is fish, but pills have benefits too. c. There is no evidence to support fish oil use with CAD. d. You can reverse CAD totally with diet and supplements.

b. The best source is fish, but pills have benefits too.

A client received tissue plasminogen activator (t-PA) after a myocardial infarction and now is on an intravenous infusion of heparin. The clients spouse asks why the client needs this medication. What response by the nurse is best? a. The t-PA didnt dissolve the entire coronary clot. b. The heparin keeps that artery from getting blocked again. c. Heparin keeps the blood as thin as possible for a longer time. d. The heparin prevents a stroke from occurring as the t-PA wears off.

b. The heparin keeps that artery from getting blocked again.

A nurse assists with the cardioversion of a client experiencing acute atrial fibrillation. Which action should the nurse take prior to the initiation of cardioversion? a. Administer intravenous adenosine. b. Turn off oxygen therapy. c. Ensure a tongue blade is available. d. Position the client on the left side.

b. Turn off oxygen therapy.

A nurse evaluates prescriptions for a client with chronic atrial fibrillation. Which medication should the nurse expect to find on this clients medication administration record to prevent a common complication of this Condition? a. Sotalol (Betapace) b. Warfarin (Coumadin) c. Atropine (Sal-Tropine) d. Lidocaine (Xylocaine)

b. Warfarin (Coumadin)

A nurse teaches a client with diabetes mellitus and a body mass index of 42 who is at high risk for coronary artery disease. Which statement related to nutrition should the nurse include in this clients teaching? a. The best way to lose weight is a high-protein, low-carbohydrate diet. b. You should balance weight loss with consuming necessary nutrients. c. A nutritionist will provide you with information about your new diet. d. If you exercise more frequently, you wont need to change your diet.

b. You should balance weight loss with consuming necessary nutrients.

The diagnosis of heart failure is usually confirmed by: a. a chest x-ray. b. an echocardiogram. c. an electrocardiogram. d. ventriculogram.

b. an echocardiogram.

A varicose vein is caused by: a. phlebothrombosis. b. an incompetent venous valve. c. venospasm. d. venous occlusion

b. an incompetent venous valve.

Raynaud's disease is a form of: a. arterial vessel occlusion caused by multiple emboli that develop in the heart and are transported through the systemic circulation. b. arteriolar vasoconstriction, usually on the fingertips, that results in coldness, pain, and pallor. c. peripheral venospasm in the lower extremities owing to valve damage resulting from prolonged venous stasis. d. phlebothrombosis related to prolonged vasoconstriction resulting from overexposure to the cold.

b. arteriolar vasoconstriction, usually on the fingertips, that results in coldness, pain, and pallor.

The most common cause of all thoracic aortic aneurysms is: a. a congenital defect in the vessel wall. b. atherosclerosis. c. infection. d. trauma

b. atherosclerosis.

A "sawtooth" P wave is seen on an ECG strip with: a. sinus bradycardia. b. atrial flutter. c. atrioventricular nodal reentry. d. premature junctional complex.

b. atrial flutter.

An antidote for propranolol hydrochloride (a beta-adrenergic blocker) that is used to treat bradycardia is: a. digoxin. b. atropine. c. protamine sulfate. d. sodium nitroprusside.

b. atropine.

To save a limb that is affected by occlusion of a major artery, surgery must be initiated before necrosis develops, which is usually: a. within the first 4 hours. b. between 6 and 10 hours. c. between 12 and 24 hours. d. within 1 to 2 days

b. between 6 and 10 hours.

The first heart sound is generated by: a. closure of the aortic valve. b. closure of the atrioventricular valves. c. opening of the atrioventricular valves. d. opening of the pulmonic valve.

b. closure of the atrioventricular valves.

The most characteristic symptom of pericarditis is: a. dyspnea. b. constant chest pain. c. fatigue lasting more than 1 month. d. uncontrolled restlessness.

b. constant chest pain.

Ermelina has nitroglycerin at her bedside to take PRN. The nurse knows that nitroglycerin acts in all of the following ways except: a. causing venous pooling throughout the body. b. constricting arterioles to lessen peripheral blood flow. c. dilating the coronary arteries to increase the oxygen supply. d. lowering systemic blood pressure.

b. constricting arterioles to lessen peripheral blood flow.

The most common heart disease for adults in the United States is: a. angina pectoris. b. coronary artery disease. c. myocardial infarction. d. valvular heart disease.

b. coronary artery disease.

The primary cause of heart failure is: a. arterial hypertension. b. coronary atherosclerosis. c. myocardial dysfunction. d. valvular dysfunction.

b. coronary atherosclerosis.

Myocardial cell damage can be reflected by high levels of cardiac enzymes. The cardiacspecific isoenzyme is: a. alkaline phosphatase. b. creatine kinase (CK-MB). c. myoglobin. d. troponin.

b. creatine kinase (CK-MB).

The nurse evaluates a series of laboratory tests within the first few hours. She knows that a positive indicator of cell damage is: a. decreased level of troponin. b. elevated creatine kinase (CK-MB). c. lower level of myoglobin. d. all of the above.

b. elevated creatine kinase (CK-MB).

Because the area of infarction develops over minutes to hours, the nurse knows to interpret the following ECG results as indicative of initial myocardial injury: a. abnormal Q waves. b. enlarged T wave. c. inverted T wave. d. ST segment depression.

b. enlarged T wave.

Ventricular bigeminy refers to a conduction defect in which: a. conduction is primarily from the AV node. b. every other beat is premature. c. rhythm is regular but fast. d. the rate is between 150 and 250 bpm.

b. every other beat is premature.

The most common nursing diagnosis for patients awaiting cardiac surgery is: a. activity intolerance. b. fear related to the surgical procedure. c. decreased cardiac output. d. anginal pain.

b. fear related to the surgical procedure.

The most frequent cause for hospitalization for people older than 75 years of age is: a. angina pectoris. b. heart failure. c. hypertension. d. pulmonary edema.

b. heart failure.

Clinical manifestations of acute venous insufficiency include all of the following except: a. cool and cyanotic skin. b. initial absence of edema. c. sharp pain that may be relieved by the elevation of the extremity. d. full superficial veins.

b. initial absence of edema.

Ermelina, a 64-year-old retired secretary, is admitted to the medical-surgical area for management of chest pain caused by angina pectoris. 1. The nurse knows that the basic cause of angina pectoris is believed to be: a. dysrhythmias triggered by stress. b. insufficient coronary blood flow. c. minute emboli discharged through the narrowed lumens of the coronary vessels. d. spasms of the vessel walls owing to excessive secretion of epinephrine (adrenaline).

b. insufficient coronary blood flow.

The most common site of myocardial infarction is the: a. left atrium. b. left ventricle. c. right atrium. d. right ventricle.

b. left ventricle.

Cardiogenic shock is pump failure that occurs primarily as the result of: a. coronary artery stenosis. b. left ventricular damage. c. myocardial ischemia. d. right atrial flutter.

b. left ventricular damage.

Buerger's disease is characterized by all of the following except: a. arterial thrombus formation and occlusion. b. lipid deposits in the arteries. c. redness or cyanosis in the limb when it is dependent. d. venous inflammation and occlusion.

b. lipid deposits in the arteries.

With peripheral arterial insufficiency, leg pain during rest can be reduced by: a. elevating the limb above heart level. b. lowering the limb so that it is dependent. c. massaging the limb after application of cold compresses. d. placing the limb in a plane horizontal to the body

b. lowering the limb so that it is dependent.

The nurse reminds Mr. Wolman to sleep with two pillows to elevate his head about 10 in. This position is recommended because: a. preload can be increased, thus enhancing cardiac output. b. pulmonary congestion can be reduced. c. venous return to the lungs can be improved, thus reducing peripheral edema. d. all of the above can help relieve his symptoms.

b. pulmonary congestion can be reduced.

According to the American College of Cardiology and the American Heart Association, a patient presenting with left ventricular dysfunction without symptoms of heart failure would be classified as: a. stage A. b. stage B. c. stage C. d. stage D

b. stage B.

The nurse auscultates the apex of the heart by placing a stethoscope over: a. Erb's point. b. the fifth intercostal space. c. the pulmonic area. d. the tricuspid area.

b. the fifth intercostal space.

The nurse is aware that ischemic tissue remains sensitive to oxygen demands, because scar formation is not seen until the: a. second week. b. third week. c. sixth week. d. eighth week

b. third week.

The nurse expects a postoperative PTCA patient to be discharged: a. the same day as surgery. b. within 24 hours of the procedure. c. 3 days later. d. after 1 week.

b. within 24 hours of the procedure.

Digitalis toxicity is a key concern in digitalis therapy. A therapeutic digitalis level should be: a. 0.25 to 0.35 mg/mL. b. 0.30 to 4.0 mg/mL c. 0.5 to 2.0 mg/mL. d. 2.5 to 4.0 mg/mL.

c. 0.5 to 2.0 mg/mL.

A client presents to the emergency department with an acute myocardial infarction (MI) at 1500 (3:00 PM). The facility has 24-hour catheterization laboratory abilities. To meet The Joint Commissions Core Measures set, by what time should the client have a percutaneous coronary intervention performed? a. 1530 (3:30 PM) b. 1600 (4:00 PM) c. 1630 (4:30 PM) d. 1700 (5:00 PM)

c. 1630 (4:30 PM)

Serial blood cultures identified Streptococcus viridans as the causative organism, and parenteral antibiotic treatment was initiated. The nurse expects that Mr. Fontana will probably remain on the antibiotic intravenous infusion for: a. 5 days. b. 1 week. c. 2 to 6 weeks. d. 8 to 10 weeks.

c. 2 to 6 weeks.

Patient education includes telling someone who takes nitroglycerin sublingually that he or she should take 1,then go quickly to the nearest emergency department if no relief has been obtained after taking ______tablet(s) at 5-minute intervals. a. 1 b. 2 c. 3 d. 4 to 5

c. 3

. The incidence of coronary artery disease tends to be equal for men and women after the age of: a. 45 years. b. 50 years. c. 55 years. d. 65 years.

c. 55 years.

A nurse assesses clients on a medical-surgical unit. Which client should the nurse identify as having the greatest risk for cardiovascular disease? a. An 86-year-old man with a history of asthma b. A 32-year-old Asian-American man with colorectal cancer c. A 45-year-old American Indian woman with diabetes mellitus d. A 53-year-old postmenopausal woman who is on hormone therapy

c. A 45-year-old American Indian woman with diabetes mellitus

A nurse performs an admission assessment on a 75-year-old client with multiple chronic diseases. The clients blood pressure is 135/75 mm Hg and oxygen saturation is 94% on 2 liters per nasal cannula. The nurse assesses the clients rhythm on the cardiac monitor and observes the reading shown below: Which action should the nurse take first? a. Begin external temporary pacing. b. Assess peripheral pulse strength. c. Ask the client what medications he or she takes. d. Administer 1 mg of atropine.

c. Ask the client what medications he or she takes.

A nurse assesses an older adult client who has multiple chronic diseases. The clients heart rate is 48 beats/min. Which action should the nurse take first? a. Document the finding in the chart. b. Initiate external pacing. c. Assess the clients medications. d. Administer 1 mg of atropine.

c. Assess the clients medications.

A nurse assesses a client 2 hours after a cardiac angiography via the left femoral artery. The nurse notes that the left pedal pulse is weak. Which action should the nurse take? a. Elevate the leg and apply a sandbag to the entrance site. b. Increase the flow rate of intravenous fluids. c. Assess the color and temperature of the left leg. d. Document the finding as left pedal pulse of +1/4.

c. Assess the color and temperature of the left leg.

A key diagnostic laboratory test for heart failure is the: a. blood urea nitrogen (BUN). b. complete blood cell count. c. B-type natriuretic peptide. d. serum electrolyte counts.

c. B-type natriuretic peptide.

The nurse asks a client who has experienced ventricular dysrhythmias about substance abuse. The client asks, Why do you want to know if I use cocaine? How should the nurse respond? a. Substance abuse puts clients at risk for many health issues. b. The hospital requires that I ask you about cocaine use. c. Clients who use cocaine are at risk for fatal dysrhythmias. d. We can provide services for cessation of substance abuse

c. Clients who use cocaine are at risk for fatal dysrhythmias.

A nurse assesses an older adult client who is experiencing a myocardial infarction. Which clinical manifestation should the nurse expect? a. Excruciating pain on inspiration b. Left lateral chest wall pain c. Disorientation and confusion d. Numbness and tingling of the arm

c. Disorientation and confusion

Which of the following findings is not a significant risk factor for heart disease? a. Cholesterol, 280 mg/dL b. LDL, 160 mg/dL c. High-density lipoproteins (HDL), 80 mg/dL d. A ratio of low-density lipoproteins (LDL) to HDL, 4.5 to 1.0

c. High-density lipoproteins (HDL), 80 mg/dL

A nurse assesses a client in an outpatient clinic. Which statement alerts the nurse to the possibility of left- sided heart failure? a. I have been drinking more water than usual. b. I am awakened by the need to urinate at night. c. I must stop halfway up the stairs to catch my breath. d. I have experienced blurred vision on several occasions.

c. I must stop halfway up the stairs to catch my breath.

Which statements by the client indicate good understanding of foot care in peripheral vascular disease? (Select all that apply.) a. A good abrasive pumice stone will keep my feet soft. b. Ill always wear shoes if I can buy cheap flip-flops. c. I will keep my feet dry, especially between the toes. d. Lotion is important to keep my feet smooth and soft. e. Washing my feet in room-temperature water is best.

c. I will keep my feet dry, especially between the toes. d. Lotion is important to keep my feet smooth and soft. e. Washing my feet in room-temperature water is best.

The provider requests the nurse start an infusion of an inotropic agent on a client. How does the nurse explain the action of these drugs to the client and spouse? a. It constricts vessels, improving blood flow. b. It dilates vessels, which lessens the work of the heart. c. It increases the force of the hearts contractions. d. It slows the heart rate down for better filling.

c. It increases the force of the hearts contractions.

A student nurse asks what essential hypertension is. What response by the registered nurse is best? a. It means it is caused by another disease. b. It means it is essential that it be treated. c. It is hypertension with no specific cause. d. It refers to severe and life-threatening hypertension.

c. It is hypertension with no specific cause.

A telemetry nurse assesses a client with third-degree heart block who has wide QRS complexes and a heart rate of 35 beats/min on the cardiac monitor. Which assessment should the nurse complete next? a. Pulmonary auscultation b. Pulse strength and amplitude c. Level of consciousness d. Mobility and gait stability

c. Level of consciousness

An example of a beta-blocker that is administered to decrease automaticity is: a. Cardizem. b. Cordarone. c. Lopressor. d. Rythmol.

c. Lopressor.

On auscultation, the nurse suspects a diagnosis of mitral valve regurgitation when which of the following is heard? a. Mitral click b. High-pitched blowing sound at the apex c. Low-pitched diastolic murmur at the apex d. Diastolic murmur at the left sternal border

c. Low-pitched diastolic murmur at the apex

A client in the cardiac stepdown unit reports severe, crushing chest pain accompanied by nausea and vomiting. What action by the nurse takes priority? a. Administer an aspirin. b. Call for an electrocardiogram (ECG). c. Maintain airway patency. d. Notify the provider

c. Maintain airway patency.

A client had an acute myocardial infarction. What assessment finding indicates to the nurse that a significant complication has occurred? a. Blood pressure that is 20 mm Hg below baseline b. Oxygen saturation of 94% on room air c. Poor peripheral pulses and cool skin d. Urine output of 1.2 mL/kg/hr for 4 hours

c. Poor peripheral pulses and cool skin

So that blood may flow from the right ventricle to the pulmonary artery, which of the following conditions is not required? a. The atrioventricular valves must be closed. b. The pulmonic valve must be open. c. Right ventricular pressure must be less than pulmonary arterial pressure. d. Right ventricular pressure must rise with systole.

c. Right ventricular pressure must be less than pulmonary arterial pressure.

A nurse cares for a client with atrial fibrillation who reports fatigue when completing activities of dailyliving. What interventions should the nurse implement to address this clients concerns? a. Administer oxygen therapy at 2 liters per nasal cannula. b. Provide the client with a sleeping pill to stimulate rest. c. Schedule periods of exercise and rest during the day. d. Ask unlicensed assistive personnel to help bathe the client.

c. Schedule periods of exercise and rest during the day.

A nurse administers prescribed adenosine (Adenocard) to a client. Which response should the nurse assess for as the expected therapeutic response? a. Decreased intraocular pressure b. Increased heart rate c. Short period of asystole d. Hypertensive crisis

c. Short period of asystole

A nurse assesses a client who is recovering after a left-sided cardiac catheterization. Which assessment finding requires immediate intervention? a. Urinary output less than intake b. Bruising at the insertion site c. Slurred speech and confusion d. Discomfort in the left leg

c. Slurred speech and confusion

A client is receiving an infusion of tissue plasminogen activator (t-PA). The nurse assesses the client to be disoriented to person, place, and time. What action by the nurse is best? a. Assess the clients pupillary responses. b. Request a neurologic consultation. c. Stop the infusion and call the provider. d. Take and document a full set of vital signs.

c. Stop the infusion and call the provider.

A nurse prepares a client for coronary artery bypass graft surgery. The client states, I am afraid I might die. How should the nurse respond? a. This is a routine surgery and the risk of death is very low. b. Would you like to speak with a chaplain prior to surgery? c. Tell me more about your concerns about the surgery. d. What support systems do you have to assist you?

c. Tell me more about your concerns about the surgery.

A nurse cares for a client who is recovering from a myocardial infarction. The client states, I will need to stop eating so much chili to keep that indigestion pain from returning. How should the nurse respond? a. Chili is high in fat and calories; it would be a good idea to stop eating it. b. The provider has prescribed an antacid for you to take every morning. c. What do you understand about what happened to you? d. When did you start experiencing this indigestion?

c. What do you understand about what happened to you?

After teaching a client who is recovering from a heart transplant to change positions slowly, the client asks, Why is this important? How should the nurse respond? a. Rapid position changes can create shear and friction forces, which can tear out your internal vascular sutures. b. Your new vascular connections are more sensitive to position changes, leading to increased intravascular pressure and dizziness. c. Your new heart is not connected to the nervous system and is unable to respond to decreases in blood pressure caused by position changes. d. While your heart is recovering, blood flow is diverted away from the brain, increasing the risk for stroke

c. Your new heart is not connected to the nervous system and is unable to respond to decreases in blood pressure caused by position changes.

The nurse assessing a patient for postural hypotension recognizes that the following is a positive sign: a. a heart rate of 5 to 20 bpm above the resting rate. b. an unchanged systolic pressure. c. an increase of 10 mm Hg reading. d. an increase of 5 mm Hg in diastolic pressure.

c. an increase of 10 mm Hg reading.

Cardioversion is used to terminate dysrhythmias. With cardioversion, the: a. amount of voltage used should exceed 400 W-s. b. electrical impulse can be discharged during the T wave. c. defibrillator should be set to deliver a shock during the QRS complex. d. above statements are all true.

c. defibrillator should be set to deliver a shock during the QRS complex.

First-degree AV block is characterized by: a. a variable heart rate, usually fewer than 60 bpm. b. an irregular rhythm. c. delayed conduction, producing a prolonged PR interval. d. P waves hidden with the QRS complex.

c. delayed conduction, producing a prolonged PR interval.

A candidate for percutaneous transluminal coronary angioplasty (PTCA) is a patient with coronary artery disease who: a. has compromised left ventricular function. b. has had angina longer than 3 years. c. has at least 70% occlusion of a major coronary artery. d. has questionable left ventricular function.

c. has at least 70% occlusion of a major coronary artery.

Ermelina took a nitroglycerin tablet at 10:00 AM, after her morning care. It did not relieve her pain, so, 5 minutes later, she repeated the dose. Ten minutes later and still in pain, she calls the nurse, who should: a. administer a PRN dose of diazepam (Valium), try to calm her, and recommend that she rest in a chair with her legs dependent to encourage venous pooling. b. assist her to the supine position, give her oxygen at 6 L/min, and advise her to rest in bed. c. help her to a comfortable position, give her oxygen at 2 L/min, and call her physician. d. suggest that she take double her previous dose after 5 minutes and try to sleep to decrease her body's need for oxygen.

c. help her to a comfortable position, give her oxygen at 2 L/min, and call her physician.

Changes in cardiac structure associated with aging would include all of the following except: a. elongation of the aorta. b. endocardial fibrosis. c. increased sensitivity to baroreceptors. d. the increased size of the left atrium.

c. increased sensitivity to baroreceptors.

The medical record lists a probable diagnosis of chronic stable angina. The nurse knows that Ermelina's pain: a. has increased progressively in frequency and duration. b. is incapacitating. c. is relieved by rest and is predictable. d. usually occurs at night and may be relieved by sitting upright

c. is relieved by rest and is predictable.

The pathophysiology of mitral stenosis is consistent with: a. aortic stenosis. b. left ventricular failure. c. left atrial hypertrophy. d. all of the above.

c. left atrial hypertrophy.

The drug of choice during cardiopulmonary resuscitation to suppress ventricular dysrhythmias is: a. atropine. b. epinephrine. c. lidocaine. d. morphine.

c. lidocaine.

Five minutes after the initial blood pressure measurement is taken, the nurse assesses additional readings with the patient in a sitting and then in a standing position. The reading indicative of an abnormal postural response would be: a. lying, 140/110; sitting, 130/110; standing, 135/106 mm Hg. b. lying, 140/110; sitting, 135/112; standing, 130/115 mm Hg. c. lying, 140/110; sitting, 135/100; standing, 120/90 mm Hg. d. lying, 140/110; sitting, 130/108; standing, 125/108 mm Hg.

c. lying, 140/110; sitting, 135/100; standing, 120/90 mm Hg.

An intravenous analgesic frequently administered to relieve chest pain associated with myocardial infarction is: a. meperidine hydrochloride. b. hydromorphone hydrochloride. c. morphine sulfate. d. codeine sulfate.

c. morphine sulfate.

The most common vasodilator used to treat myocardial pain is: a. amyl nitrite. b. Inderal. c. nitroglycerine. d. Pavabid HCl.

c. nitroglycerine.

The most outstanding symptom of Buerger's disease is: a. a burning sensation. b. cramping in the feet. c. pain. d. paresthesia.

c. pain.

Mr. Anderson is a 45-year-old executive with a major oil firm. Lately he has experienced frequent episodes of chest pressure that are relieved with rest. He has requested a complete physical examination. The nurse conducts an initial cardiac assessment. 1. The nurse immediately inspects the patient's skin. She observes a bluish tinge around the patient's lips. She knows that this is an indication of: a. central cyanosis. b. pallor. c. peripheral cyanosis. d. xanthelasma

c. peripheral cyanosis.

A nonmodifiable risk factor for atherosclerosis is: a. stress. b. obesity. c. positive family history. d. hyperlipidemia.

c. positive family history.

Extremity paresthesia, dysrhythmias (peaked T waves), and mental confusion after cardiac surgery are signs of electrolyte imbalance related to the level of: a. calcium. b. magnesium. c. potassium. d. sodium.

c. potassium.

A clinical manifestation of pericardial effusion is: a. widening pulse pressure. b. a decrease in venous pressure. c. shortness of breath. d. an increase in blood pressure.

c. shortness of breath.

A nurse should teach a patient with chronic venous insufficiency to do all of the following except: a. avoid constricting garments. b. elevate the legs above the heart level for 30 minutes every 2 hours. c. sit as much as possible to rest the valves in the legs. d. sleep with the foot of the bed elevated about 6 in.

c. sit as much as possible to rest the valves in the legs.

Probably the most helpful diagnostic test to identify cardiomyopathy is: a. serial enzyme studies. b. cardiac catheterization. c. the echocardiogram. d. the phonocardiogram.

c. the echocardiogram.

Mr. Wolman is to be discharged from the hospital to home. He is 79 years old, lives with his wife, and has just recovered from mild pulmonary edema secondary to congestive heart failure. 1. The most common cause of pulmonary edema is: ________________________________________________. .

cardiac disease

Even with successful treatment, organ damage can occur. Cardiac complications may include: ______________________, ______________________, ______________________, and ______________________.

congestive heart failure, strokes, valvular stenosis, and myocardial erosion

The physician prescribed a Tegapore dressing to treat a venous ulcer. The nurse knows that the anklebrachial index (ABI) must be _______ for the circulatory status to be adequate. a. 0.10 b. 0.25 c. 0.35 d. 0.50

d. 0.50

The nurse takes a baseline blood pressure measurement after the patient has rested for 10 minutes in a supine position. The reading that reflects a reduced pulse pressure is: a. 140/90 mm Hg. b. 140/100 mm Hg. c. 140/110 mm Hg. d. 140/120 mm Hg.

d. 140/120 mm Hg.

When caring for a patient who has started anticoagulant therapy with warfarin (Coumadin), the nurse knows not to expect therapeutic benefits for: a. at least 12 hours. b. the first 24 hours. c. 2 to 3 days. d. 3 to 5 days.

d. 3 to 5 days.

The nurse returns Mr. Anderson to the supine position and measures for jugular vein distention. The finding that would initially indicate an abnormal increase in the volume of the venous system would be obvious distention of the veins with the patient at what angle? a. 15 degrees b. 25 degrees c. 35 degrees d. 45 degrees

d. 45 degrees

Atrial fibrillation is associated with a heart rate up to: a. 300 bpm. b. 400 bpm. c. 500 bpm. d. 600 bpm

d. 600 bpm

An emergency department nurse triages clients who present with chest discomfort. Which client should the nurse plan to assess first? a. A 42-year-old female who describes her pain as a dull ache with numbness in her fingers b. A 49-year-old male who reports moderate pain that is worse on inspiration c. A 53-year-old female who reports substernal pain that radiates to her abdomen d. A 58-year-old male who describes his pain as intense stabbing that spreads across his chest

d. A 58-year-old male who describes his pain as intense stabbing that spreads across his chest

A nurse assesses a client after administering isosorbide mononitrate (Imdur). The client reports a headache. Which action should the nurse take? a. Initiate oxygen therapy. b. Hold the next dose of Imdur. c. Instruct the client to drink water. d. Administer PRN acetaminophen.

d. Administer PRN acetaminophen.

. A candidate for coronary artery bypass grafting (CABG) must meet which of the following criteria? a. A blockage that cannot be treated by PTCA b. Greater than 60% blockage in the left main coronary artery. c. Unstable angina. d. All of the above.

d. All of the above.

A nurse assesses a client who is scheduled for a cardiac catheterization. Which assessment should the nurse complete prior to this procedure? a. Clients level of anxiety b. Ability to turn self in bed c. Cardiac rhythm and heart rate d. Allergies to iodine-based agents

d. Allergies to iodine-based agents

A nurse teaches a client who is prescribed digoxin (Lanoxin) therapy. Which statement should the nurse include in this clients teaching? a. Avoid taking aspirin or aspirin-containing products. b. Increase your intake of foods that are high in potassium. c. Hold this medication if your pulse rate is below 80 beats/min. d. Do not take this medication within 1 hour of taking an antacid.

d. Do not take this medication within 1 hour of taking an antacid.

A nurse prepares to defibrillate a client who is in ventricular fibrillation. Which priority intervention should the nurse perform prior to defibrillating this client? a. Make sure the defibrillator is set to the synchronous mode. b. Administer 1 mg of intravenous epinephrine. c. Test the equipment by delivering a smaller shock at 100 joules. d. Ensure that everyone is clear of contact with the client and the bed.

d. Ensure that everyone is clear of contact with the client and the bed.

The nurse expects that positive inotropic medications would be administered to treat cardiogenic shock, with the exception of: a. Adrenalin. b. Dobutrex. c. Intropin. d. Levophed.

d. Levophed.

A nurse is assessing a client who had a myocardial infarction. Upon auscultating heart sounds, the nurse hears the following sound. What action by the nurse is most appropriate? (Click the media button to hear the audio clip.) a. Assess for further chest pain. b. Call the Rapid Response Team. c. Have the client sit upright. d. Listen to the clients lung sounds.

d. Listen to the clients lung sounds.

An older client with peripheral vascular disease (PVD) is explaining the daily foot care regimen to the family practice clinic nurse. What statement by the client may indicate a barrier to proper foot care? a. I nearly always wear comfy sweatpants and house shoes. b. Im glad I get energy assistance so my house isnt so cold. c. My daughter makes sure I have plenty of lotion for my feet. d. My hands shake when I try to do things requiring coordination.

d. My hands shake when I try to do things requiring coordination.

A nursing student is caring for a client with an abdominal aneurysm. What action by the student requires the registered nurse to intervene? a. Assesses the client for back pain b. Auscultates over abdominal bruit c. Measures the abdominal girth d. Palpates the abdomen in four quadrants

d. Palpates the abdomen in four quadrants

A student nurse is assessing the peripheral vascular system of an older adult. What action by the student would cause the faculty member to intervene? a. Assessing blood pressure in both upper extremities b. Auscultating the carotid arteries for any bruits c. Classifying capillary refill of 4 seconds as normal d. Palpating both carotid arteries at the same time

d. Palpating both carotid arteries at the same time

The nurse is preparing to change a clients sternal dressing. What action by the nurse is most important? a. Assess vital signs. b. Don a mask and gown. c. Gather needed supplies. d. Perform hand hygiene.

d. Perform hand hygiene.

A client undergoing hemodynamic monitoring after a myocardial infarction has a right atrial pressure of 0.5 mm Hg. What action by the nurse is most appropriate? a. Level the transducer at the phlebostatic axis. b. Lay the client in the supine position. c. Prepare to administer diuretics. d. Prepare to administer a fluid bolus.

d. Prepare to administer a fluid bolus.

A nurse assesses a client after administering a prescribed beta blocker. Which assessment should the nurse expect to find? a. Blood pressure increased from 98/42 mm Hg to 132/60 mm Hg b. Respiratory rate decreased from 25 breaths/min to 14 breaths/min c. Oxygen saturation increased from 88% to 96% d. Pulse decreased from 100 beats/min to 80 beats/min

d. Pulse decreased from 100 beats/min to 80 beats/min

A nurse is preparing a client for a femoropopliteal bypass operation. What actions does the nurse delegate to the unlicensed assistive personnel (UAP)? (Select all that apply.) a. Administering preoperative medication b. Ensuring the consent is signed c. Marking pulses with a pen d. Raising the siderails on the bed e. Recording baseline vital signs

d. Raising the siderails on the bed e. Recording baseline vital signs

A nurse assesses a clients electrocardiogram (ECG) and observes the reading shown below: How should the nurse document this clients ECG strip? a. Ventricular tachycardia b. Ventricular fibrillation c. Sinus rhythm with premature atrial contractions (PACs) d. Sinus rhythm with premature ventricular contractions (PVCs)

d. Sinus rhythm with premature ventricular contractions (PVCs)

A nurse is caring for a client with acute pericarditis who reports substernal precordial pain that radiates to the left side of the neck. Which nonpharmacologic comfort measure should the nurse implement? a. Apply an ice pack to the clients chest. b. Provide a neck rub, especially on the left side. c. Allow the client to lie in bed with the lights down. d. Sit the client up with a pillow to lean forward on

d. Sit the client up with a pillow to lean forward on

A client is in the preoperative holding area prior to an emergency coronary artery bypass graft (CABG). The client is yelling at family members and tells the doctor to just get this over with when asked to sign the consent form. What action by the nurse is best? a. Ask the family members to wait in the waiting area. b. Inform the client that this behavior is unacceptable. c. Stay out of the room to decrease the clients stress levels. d. Tell the client that anxiety is common and that you can help.

d. Tell the client that anxiety is common and that you can help.

The nurse is reviewing the lipid panel of a male client who has atherosclerosis. Which finding is most concerning? a. Cholesterol: 126 mg/dL b. High-density lipoprotein cholesterol (HDL-C): 48 mg/dL c. Low-density lipoprotein cholesterol (LDL-C): 122 mg/dL d. Triglycerides: 198 mg/dL

d. Triglycerides: 198 mg/dL

A nurse cares for a client who has advanced cardiac disease and states, I am having trouble sleeping at night. How should the nurse respond? a. I will consult the provider to prescribe a sleep study to determine the problem. b. You become hypoxic while sleeping; oxygen therapy via nasal cannula will help. c. A continuous positive airway pressure, or CPAP, breathing mask will help you breathe at night. d. Use pillows to elevate your head and chest while you are sleeping.

d. Use pillows to elevate your head and chest while you are sleeping.

A nurse assesses a clients electrocardiograph tracing and observes that not all QRS complexes are preceded by a P wave. How should the nurse interpret this observation? a. The client has hyperkalemia causing irregular QRS complexes. b. Ventricular tachycardia is overriding the normal atrial rhythm. c. The clients chest leads are not making sufficient contact with the skin. d. Ventricular and atrial depolarizations are initiated from different sites.

d. Ventricular and atrial depolarizations are initiated from different sites.

. A nurse teaches a client who has a history of heart failure. Which statement should the nurse include in this clients discharge teaching? a. Avoid drinking more than 3 quarts of liquids each day. b. Eat six small meals daily instead of three larger meals. c. When you feel short of breath, take an additional diuretic. d. Weigh yourself daily while wearing the same amount of clothing.

d. Weigh yourself daily while wearing the same amount of clothing.

A nurse cares for a client with end-stage heart failure who is awaiting a transplant. The client appears depressed and states, I know a transplant is my last chance, but I dont want to become a vegetable. How should the nurse respond? a. Would you like to speak with a priest or chaplain? b. I will arrange for a psychiatrist to speak with you. c. Do you want to come off the transplant list? d. Would you like information about advance directives?

d. Would you like information about advance directives?

A significant cause of venous thrombosis is: a. altered blood coagulation. b. stasis of blood. c. vessel wall injury. d. all of the above

d. all of the above

In health teaching, the nurse should suggest methods to increase arterial blood supply, which include: a. a planned program involving systematic lowering of the extremity below heart level. b. Buerger-Allen exercises. c. graded extremity exercises. d. all of the above

d. all of the above

Pulmonary edema is characterized by: a. elevated left ventricular end-diastolic pressure. b. a rise in pulmonary venous pressure. c. increased hydrostatic pressure. d. all of the above alterations

d. all of the above alterations

Stroke volume of the heart is determined by: a. the degree of cardiac muscle strength (precontraction). b. the intrinsic contractility of the cardiac muscle. c. the pressure gradient against which the muscle ejects blood during contraction. d. all of the above factors.

d. all of the above factors.

. Lumen narrowing with atherosclerosis is caused by: a. atheroma formation on the intima. b. scarred endothelium. c. thrombus formation. d. all of the above.

d. all of the above.

. Mr. Fontana needs to be advised that prophylactic antibiotic therapy is also recommended for: a. tooth extraction. b. bronchoscopy. c. cystoscopy. d. all of the above.

d. all of the above.

. The need for surgical intervention in coronary artery disease (CAD) is determined by the: a. amount of stenosis in the coronary arteries. b. myocardial area served by the stenotic artery. c. occurrence of previous infarction related to the affected artery. d. all of the above.

d. all of the above.

A nurse who suspects the presence of an abdominal aortic aneurysm should look for the presence of: a. a pulsatile abdominal mass. b. low back pain. c. lower abdominal pain. d. all of the above.

d. all of the above.

Clinical manifestations of deep vein obstruction include: a. edema and limb pain. b. ankle engorgement. c. leg circumference differences. d. all of the above.

d. all of the above.

Clinical manifestations of infective endocarditis may include: a. embolization. b. focal neurologic lesions. c. heart murmurs. d. all of the above.

d. all of the above.

Diagnosis of a thoracic aortic aneurysm is done primarily by: a. computed tomography. b. transesophageal echocardiography. c. x-ray. d. all of the above.

d. all of the above.

Initial nursing intervention includes maintenance of bed rest until the following symptom disappears: a. fever. b. friction rub. c. pain. d. all of the above.

d. all of the above.

Morphine is given in acute pulmonary edema to redistribute the pulmonary circulation to the periphery by decreasing: a. peripheral resistance. b. pulmonary capillary pressure. c. transudation of fluid. d. all of the above.

d. all of the above.

Mr. Wolman also takes Lasix (40 mg) twice a day. He is aware of signs related to hypokalemia and supplements his diet with foods high in potassium, such as: a. bananas. b. raisins. c. orange juice. d. all of the above.

d. all of the above.

Mr. Wolman takes 0.25 mg of digoxin once a day. The nurse should tell him about signs of digitalis toxicity, which include: a. anorexia. b. bradycardia and tachycardia. c. nausea and vomiting. d. all of the above.

d. all of the above.

Nursing measures to promote a clean leg ulcer include: a. applying wet-to-dry saline solution dressings, which would remove necrotic debris when changed. b. flushing out necrotic material with hydrogen peroxide. c. using an ointment that would treat the ulcer by enzymatic debridement. d. all of the above.

d. all of the above.

Postcatheterization nursing measures for a patient who has had a cardiac catheterization include: a. assessing the peripheral pulses in the affected extremity. b. checking the insertion site for hematoma formation. c. evaluating temperature and color in the affected extremity. d. all of the above.

d. all of the above.

The nurse needs to be alert to assess for clinical symptoms of possible postoperative complications of PTCA, which include: a. abrupt closure of the artery. b. arterial dissection. c. coronary artery vasospasm. d. all of the above.

d. all of the above.

With pulmonary edema, there is usually an alteration in: a. afterload. b. contractility. c. preload. d. all of the above.

d. all of the above.

The nurse needs to teach the patient with an automatic ICD that he or she must: a. avoid magnetic fields such as metal detection booths. b. call for emergency assistance if he or she feels dizzy. c. record events that trigger a shock sensation. d. be compliant with all of the above.

d. be compliant with all of the above.

The most common valvuloplasty procedure is the: a. balloon valvuloplasty. b. annuloplasty. c. chordoplasty. d. commissurotomy

d. commissurotomy

Exercise stress testing is a noninvasive procedure that can be used to assess certain aspects of cardiac function. After the test, the patient is instructed to: a. rest for a time. b. avoid stimulants. c. avoid extreme temperature changes. d. do all of the above.

d. do all of the above.

Nursing measures in hemodynamic monitoring include assessing for localized ischemia caused by inadequate arterial flow. The nurse should: a. assess the involved extremity for color temperature. b. check for capillary filling. c. evaluate pulse rate. d. do all of the above.

d. do all of the above.

The nurse advises Mr. Wolman to rest frequently at home. This advice is based on the knowledge that rest: a. decreases blood pressure. b. increases the heart reserve. c. reduces the work of the heart. d. does all of the above.

d. does all of the above.

Candidates for implantable cardioverter defibrillation (ICD) are patients at high risk who have: a. experienced syncope secondary to ventricular tachycardia. b. survived sudden cardiac death. c. sustained ventricular tachycardia. d. experienced one or more of the above

d. experienced one or more of the above

The intrinsic pacemaker rate of ventricular myocardial cells is: a. more than 80 bpm. b. 60 to 80 bpm. c. 40 to 60 bpm. d. fewer than 40 bpm.

d. fewer than 40 bpm.

When assessing vital signs in a patient with a permanent pacemaker, the nurse needs to know the: a. date and time of insertion. b. location of the generator. c. model number. d. pacer rate.

d. pacer rate.

The heart is under the control of the autonomic nervous system. Stimulation of the parasympathetic system results in all of the following except: a. slowed heart rate. b. lowered blood pressure. c. reduction in the force of contraction. d. positive inotropy.

d. positive inotropy.

Incomplete closure of the tricuspid valve results in a backward flow of blood from the: a. aorta to the left ventricle. b. left atrium to the left ventricle. c. right atrium to the right ventricle. d. right ventricle to the right atrium.

d. right ventricle to the right atrium.

The pacemaker for the entire myocardium is the: a. atrioventricular junction. b. bundle of His. c. Purkinje fibers. d. sinoatrial node.

d. sinoatrial node.

Characteristics of sinus bradycardia include all of the following except: a. a P wave precedes every QRS complex. b. every QRS complex is normal. c. the rate is 40 to 60 bpm. d. the rhythm is altered.

d. the rhythm is altered.

The most important factor in regulating the caliber of blood vessels, which determines resistance to flow, is: a. hormonal secretion. b. independent arterial wall activity. c. the influence of circulating chemicals. d. the sympathetic nervous system.

d. the sympathetic nervous system.

Heart rate is stimulated by all of the following except: a. excess thyroid hormone. b. increased levels of circulating catecholamines. c. the sympathetic nervous system. d. the vagus nerve

d. the vagus nerve

A conduction abnormality whereby no atrial impulse travels through the AV node is known as: a. first-degree AV block. b. second-degree AV block, type 1. c. second-degree AV block, type 2. d. third-degree AV block.

d. third-degree AV block.

Knowing the most serious complication of venous insufficiency, the nurse would assess the patient's lower extremities for signs of: a. rudor. b. cellulitis. c. dermatitis. d. ulceration.

d. ulceration.

A recommended position for a patient in acute pulmonary edema is: a. prone, to encourage maximum rest, thereby decreasing respiratory and cardiac rates. b. semi-Fowler's, to facilitate breathing and promote pooling of blood in the sacral area. c. Trendelenburg, to drain the upper airways of congestion. d. upright with the legs down, to decrease venous return.

d. upright with the legs down, to decrease venous return.

List the two major expected patient outcomes for nursing management of a patient with pericarditis: _________________________________________ and ______________________________________________. .

freedom from pain and absence of complications

The nurse also assesses for central nervous system manifestations of the infectious disease. She looks for symptoms such as: ______________________, ______________________, ______________________, and ______________________.

headache, transient cerebral ischemia, focal neurologic lesions, and strokes

The pain is due to the irritation of nerve endings by the buildup of ____________________ and __________

muscle metabolites and lactic acid


Set pelajaran terkait

Prep U Chapter 34: Assessment and Management of Patients with Inflammatory Rheumatic Disorders

View Set

Mod 08: Nation Building in the Americas

View Set

Managing Anxiety and Delivering your Speech

View Set

IB Economics SL - T2: Macroeconomics

View Set